You are on page 1of 26

Marathon - I

CLAT Classroom Prep Material


Part of the most Comprehensive Classroom Training, Prep Content & Test Series across the Nation.
From the producers of A.I.R. 2, 3 and 5 from Classroom Contact Program in 2019
MARATHON SESSION: CRITICAL REASONING I
QUESTIONS FOR CLASSROOM DISCUSSION

ANSWER KEY AND EXPLANATIONS


(Section-I) 3. (B); the manager explains that installing the computer
1. (B); two important facts are presented within this network during the day will disrupt work, but
question: (1) the interior of the cave is now installing during night will incur higher installations
completely filled with water, and (2) the cave charges. The manager concludes that since saving
contains stalagmites, which are formed when drops money is important, the network should be installed
of water fall to the cave floor. One might infer that in during the day. The manager assumes that the lost
a cave completely filled with water, it would be productivity during the daytime installation costs
difficult for drops of water to hit the floor. The less than the extra money the night installation
question allows us to infer, then, that the conditions would. Since you are asked what the manager
must have been different in the cave when the assumes, you should look for that in an option.
stalagmites were formed. Option (A) this claim is Option (A): Since the monetary value of the network
not supported by the information in the question. must be covered in either case, it is not an
The only inference that can be properly drawn is immediate consideration when discussing the
about the water level within the cave, not that of the proper installation time. This choice attempts to get
Mediterranean Sea as a whole. Also, the you to quickly "bite" on the cost of another factor,
information in the question suggests that the water but gives you the wrong factor, so this choice is
level is higher now than it was in the past, not lower wrong. Option (B): This is the correct option. The
as in this answer. Option (B) is the correct option. manager presumes that there is less loss from
For the stalagmites to form, water drops must have interrupting work than there is loss from installing at
been able to fall to the cave floor. To allow for this night. Option (C): Since you already know the
to happen, the water level in the cave must have relative cost of installation itself, it does not matter
been lower than its current level, at which it is whether that loss is incurred because of differences
"completely filled." Option(C) there is no indication in the installation crew. "Installation charges" clearly
that this is true. Just because they "found" the implies that the company hires the job out (your
tunnel does not mean that no one else could have employees receive wages, you do not pay them
"found" it before them. Option (D) there is no charges), so you should not have presumed that
evidence of this in the question. There is no reason this option addresses whether the company will
given that the cave must have been accessible have to hire more employees. Besides, even if the
without using the underwater tunnel. Option (E) company did have to hire more employees, that
there is nothing in the question to support this. If could still be a temporary arrangement that would
stalagmites are no longer forming, the question not cost as much as paying for a night installation.
suggests that it is because water drops can no Option (D): The question concerns when the
longer fall to the floor because the cave is flooded. network should be installed, and this choice
2. (A); in the given question we have to weaken the claim concerns what will immediately follow installation.
supported by Director of the Rexx Pharmaceutical This choice is irrelevant and incorrect. Alternately,
Company. The claim is - that the development this choice does not establish that the increased
costs for new vaccines that the health department productivity that might follow on the installation day
has requested should be subsidized by the would make up for the productivity lost while the
government, since the marketing of vaccines system was being installed. Option (E): The
promised to be less profitable than the marketing of manager does not need to assume that most of the
any other pharmaceutical product. Option (A) is the employees will be able to be productive during the
correct answer as it is not about per person one installation. Even if none of the employees are
time use of the vaccine rather it is about more productive during the installation, the extra cost of
number of people using it. Option (B) draws a installing at night could be so high that it is better to
comparison between medicines and vaccines which simply give up a day of productivity, so the
don’t weaken the argument. Option (C),(D) and (E) argument can be true even if this unnecessary
cannot be deduced from the passage. assumption is false.

Head Office: 127, Zone II, MP Nagar, Bhopal |+91-9111555433| www.legaledge.in Page 1 of 26
Marathon - I
4. (B); premise: Marijuana can inactivate herpesviruses. potential of new evidence, and the principle that the
Premise: Inactivated herpesviruses can convert risk of losing evidence for future theories outweighs
healthy cells to cancerous. Conclusion: Marijuana the advantages of displaying the results of theories
can cause cancer. The question requires that we based on current evidence speaks to that issue.
seek the option which would cause one to question You cannot plausibly deny that restoration and
the author's conclusion. Option (A): This option public access incurs some risk to evidence.
certainly doesn't weaken the argument in the 6. (D); the argument is that nesting boxes are actually
question. If anything this choice strengthens the harmful to the reproduction of wood ducks, since
author's conclusion, lending more credibility to the the easy visibility of the boxes makes them
cited results. Option (B): This is the correct option. vulnerable to the natural habit wood ducks have of
If there exists other ingredients in marijuana which laying eggs in the nests of other wood ducks. The
neutralize the harmful effects, then this of course behavior is normally harmless because real nests
weakens the conclusion that marijuana causes are hard to find. However, since the boxes are easy
cancer. Option (C): This option strengthens the to spot, the nests become overly full of eggs which
assertion that marijuana use increases the risk of somehow inhibit hatching. Option (A): The question
cancer, so this option is incorrect. Option (D): The does not state the preference of wood ducks, and
effects that might be associated with a modified you should not assume that creatures such as
THC play no role in assessing whether or not wood ducks have the ability to realize that their
marijuana increases cancer risk. Option (E): the actions will be ineffective in the long-term.
fact that it has been recommended for cancer Furthermore, it seems that wood ducks definitely
patients make it nor more or less likely that are choosing the boxes, and you should not
marijuana use causes cancer, so this option is assume that is because of a lack of natural sites, so
incorrect. this choice is unsupported and incorrect. Option
5. (E ); the archaeologist grants that the restoration project (B): Since overloading a nest with eggs actually will
the corporation offers to fund would conform to actually limit the viability of the eggs, it seems that
current theories about the site, but on the basis that this option might be false. The most successful
many parts of the site contain unexamined ducks would probably be the ones that protect their
information the archaeologist concludes that the own nests, and this choice is incorrect. Option (C):
offer should be rejected. The archaeologist is You should not infer that because the nests overfill,
probably concerned that the new evidence could they are smaller than natural sites. The nests
have a significant impact on the theories about how overfill because they are highly visible, and this
the site appeared. The archaeologist seems to choice is incorrect. Option (D): This is the correct
operate on the broad assumption that one should option. Since the main problem with the nests is
not reconstruct archaeological sites until as much that they are too visible, it makes sense that limiting
evidence as possible is evaluated. Since you are their visibility would make them a better tool. Option
asked to find the principle that would justify the (E): Since the question never discussed the
argument, you should locate a choice that destruction of any habitat, this response is
reinforces the archaeologist's broad assumption. unsupported and incorrect. Furthermore, even if the
Option (A): The archaeologist does not seem to boxes were helpful, there would be no reason to
have a problem with commercial interests; the assume they are necessary.
archaeologist is merely concerned with accuracy. 7. (E); the key to answering this question quickly and
Furthermore, even if the question clearly indicated efficiently is to understand the conditional
(it does not) that the corporation would own the site, relationships that underpin this argument. Quite
this choice does not respond to the argument, simply, any "new" theme or idea is always and
which is based on the idea of evidence, not without exception a variation on a previous theme
ownership. Option (B): This response reinforces the or idea: Creation of "new" themes/ideas →
idea that restorations should represent the most Variation of a previous theme/idea. Since the
ancient period of a site's history, but the question question stem is asking us to find the one answer
promotes the idea that the restorations should that is not necessarily true (each must be true
represent the site at the relevant civilization's EXCEPT), any answer that is logically inferable
height. Option (C): This response suggests that the from the conditional relationship above will be
corporation is wrong to make a judgment incorrect. Option (A): This option is simply the
concerning the height of the civilization in question, contrapositive of the first sentence in the question:
but the archaeologist actually seems to accept that since the crux of creativity resides in (i.e. requires)
judgment and only argues that more evidence the ability to manufacture variations on a theme, a
should be evaluated, presumably to paint a better lack of ability to manufacture such variations
picture of that civilization at its height. Option (D): connotes a lack of creativity. Because this answer
Since the archaeologist's argument was concerned is inferable from the information in the question, it is
with the state of evidence rather than the source of incorrect. Option (B): This option paraphrases the
funding, the idea that the corporation should not be main idea of the question: if each idea is a variation
the source of funding is irrelevant to the argument, on a previous one, then no idea is entirely
and this choice is wrong even if you incorrectly independent of others. Because this answer is also
presume it supports the archaeologist's conclusion. inferable from the information in the question, it is
In fact, this choice does not even support the incorrect. Option (C): The third sentence in the
conclusion, because it is entirely possible that the question suggests that careful analysis can make
corporation in question actually has a true concern us understand that every new theme or discovery is
for the site's history. Option (E): This is the correct itself always and without exception some sort of
option. The archaeologist is concerned with the variation of previous themes. Option (C) is a mere
Head Office: 127, Zone II, MP Nagar, Bhopal |+91-9111555433| www.legaledge.in Page 2 of 26
Marathon - I
paraphrase of this observation. Option (D): Quite the premise. Option (D): This option is incorrect
logically, if every idea, theme or discovery is always because it is dealing with recovering from sickness,
a variation on a theme, it follows that everyone which has nothing to do with actually getting sick or
making a discovery must be capable of developing distaste for food. Option (E): This option
manufacturing a variation on a theme. Because this is incorrect because it focuses on children refusing
option is logically supported by the question, it is to eat unfamiliar foods. If a child has never tasted a
incorrect. Option (E): This is the correct option. If all food, he or she cannot develop a distaste for that
of the statements in the question are true and every food, so this option is unrelated to our argument
scientific discovery is a variation on a previous about sickness and distaste for certain foods in
theme, then option (E) must be false as it suggests children after they have eaten them.
that some discoveries are not variations on a 10. (C); The correct option is (C). In this question the author
theme. Such a possibility is clearly ruled out by the presents two things that are correlated, and then
evidence presented in the question. Therefore, this draws a causal conclusion from them. This
option is correct. argument is a classic fallacy. Just because there is
8. (B) ; the argument concludes that since calls are the a higher percentage of red cars involved in
only means of the bats locating each other, and the accidents, this does not mean that the red cars
pups cannot distinguish between various adult cause the accidents. For the conclusion to follow,
calls, the bat mothers must be able to distinguish the argument must demonstrate that the red color
their pup's calls. The argument proceeds by actually causes accidents, but it only demonstrates
explaining what the options are, and then a correlation between red and accidents. Read this
eliminating all but one. You should focus on that question and react to it with your own
when describing what the argument seeks to do. commonsense; can getting rid of red cars really
Option (A) Since the argument does not involve prevent accidents? Attack the conclusion. Option
comparison between a sample and a population, it (A): This option is incorrect because the argument
is unlikely that the argument generalizes from a does not claim that insurance companies have or
small group, and this choice is wrong. Furthermore, do not have the right to charge higher premiums. It
this choice can be seen as somewhat contradictory, simply mentions these rates as a premise to
as pups seem different from the mothers. Option support its causal conclusion. This answer is not a
(B): This is the correct option. The argument part of the argument; therefore, it cannot be a flaw
attempts to arrive at a final explanation by in the argument. Option (B): This option is incorrect
eliminating options. Option (C) Since the argument because it also deals with the higher premiums. If
does not describe the mechanism by which this fact were true, then it may justify the higher
mothers use calls to locate their pups, this choice is premiums for red cars. However, the argument has
wrong. Option (D) Since the argument implies that already justified these premiums in a different way,
the pups and mothers may be different in their by stating that the red color of the car causes
ability to distinguish calls, it does not follow that accidents. Therefore, this answer is unrelated to the
argument makes conclusions based on similarities. argument. Option (C): This is the correct
Option (E) The argument attempts to resolve the option. Now, the causal connection between red
general rule concerning how pups and mothers color and accidents has been destroyed. If bad
reunite, not to apply that rule. drivers preferred, then they are the cause of the
9. (C); In this question the author presents only one accidents, not the red color. Therefore, if the red
premise and then draws a causal conclusion. cars were banned, these drivers would just have to
People develop distaste for foods that make them buy other cars and lives would not be saved or
sick is the only premise. This explains why children accidents reduced. Option (D): This option is
don't like some foods. There is a completely new incorrect because it tries to tempt you by naming
element in the conclusion: children. This element something that is missing in the argument: an exact
needs to be connected to the premise in order to percentage. However, in this argument we do not
strengthen the causal argument. There is also an need to know an exact percentage; it is irrelevant.
element in the premise, sickness, that is not As long as there is "higher" percentage of red cars,
present in the conclusion. We need to connect the correlation between red cars and accidents
these two lonely elements of the argument. If this exists. The flaw in the argument is mistaking this
were a weaken question, we would attack this gap correlation for a cause and effect relationship.
in the reasoning. Option (E): This option is incorrect, but it is tricky.
Option (A): This option is incorrect because if The argument does make a big assumption in the
actually weakens the argument. If children are more conclusion, jumping from automobile accidents to
likely to get food without distinctive flavor, they are lives being saved. However, the argument does not
less likely to develop aversions. Option (B): This assume that every accident results in loss of life, as
option is incorrect because it is entirely unrelated to this option states. Also, the flaw is not in this
the question. If anything, this answer offers another assumption, but in the cause and effect conclusion.
reason for children to develop aversions: they don't If you attack the question as you read it, you should
see the health benefits of some foods so they go by see it.
taste alone. Option (C): This is the correct option. It 11. (D); In this question a typical credit card scenario is
fills in the gap that the question is missing. Children presented and then a conclusion drawn. Again, pay
tend to have more acute taste and become sick, careful attention to the conclusion: "customers who
which connects the element in the premise, purchase merchandise [by mail] spend less" than
sickness, with the new element in the conclusion, they would spend if they went to a store. Although
children. If children are more likely to become sick, the premises demonstrate that the "purchase price"
they are more likely to develop distaste, based on is lower by mail, they never mention the idea of how
Head Office: 127, Zone II, MP Nagar, Bhopal |+91-9111555433| www.legaledge.in Page 3 of 26
Marathon - I
much the customer actually spends, which is a new 14. (C); the correct option is (C). In this question, Yolanda
element in the conclusion. This demonstrates the presents an argument and then Arjun refutes it by
weakness of the argument, and where a Defender presenting an example. Yolanda presents
assumption might be necessary. We are looking for similarities between computer manipulation and
an option that, when we negate it, destroys the joyriding but then points out a difference between
conclusion of the argument. Option (A): This option, them. Arjun disagrees with that difference by
when negated, basically says that other credit-card presenting a hypothetical situation. Arjun's
companies offer the same merchandise at lower conclusion is that "computer crimes also cause
prices. This does not affect the conclusion one way physical harm to people." However, he only
or another since it deals only with the comparison presents a hypothetical situation and does not show
between items offered by this particular company that these crimes actually cause harm, as he
by mail and those items in a retail store. Option (B): asserts. This is the flaw in his argument. Option (A)
This option, when negated, only says that Arjun tries to demonstrate that the distinction made
customers can buy merchandise not available in in Yolanda's argument is inaccurate, so this option
stores. It does not affect the conclusion because it is not a flaw in his reasoning but what his argument
says nothing about money being spent or prices. tries (but fails) to accomplish. Option (B) Although
Option (C): This option, when negated, says the Arjun does deny Yolanda's conclusion, he also
companies make customers accrue a large number does provide evidence, i.e. his hypothetical
of points, but still reinforces the fact that prices are situation. Option (C): This is the correct option. For
lower. When negated, it does not destroy the Arjun's conclusion to be true, computer crimes must
conclusion. Option (D): This is the correct option. cause harm to people. He has only shown that they
Notice that it deals specifically with the gap in the could possibly cause harm in one hypothetical
reasoning – spending. When negated, this option circumstance; he has not shown that they do in
destroys the conclusion because, even though the reality. Option (D) this option employs the word
prices may be lower, the customer spends more "necessary" to try to tempt you, but does not apply
when shipping is added. Thus, this option is to Arjun's argument in any way. Option (E) Arjun's
necessary for the conclusion to follow. Option (E): evidence is consistent with his conclusion, so this is
This option, when negated, does not affect the not the correct answer.
question in any way. It introduces the new element 15. (C); option (A) mentions a situation which is not
of the "manufacturers' suggested retail price," which reasoning for our condition. Option (B) talks about
is irrelevant to the argument in the question. number of years which is an irrelevant criterion for
12. (D); this question denies the relationship between the consideration. Option (D) and (E) are out of context.
development of colic in infants and the consumption Option (C) is the correct answer as this reasoning
of cow’s milk, saying that the infants’ drinking of can appropriately answer the above the criticism.
cow’s milk likely is not the cause of the 16. (A); the question begins by contrasting similar stars--
development of colic. The author supports this brown dwarfs and red dwarfs. Brown dwarfs are too
claim by stating that infants fed breast milk cool, whereas red dwarfs are just hot enough, to
exclusively still occasionally develop colic. To harm burn oxygen. That particular contrast is actually
this conclusion you should look for an option that irrelevant to the argumentation, which takes place
shows that there is in fact some connection after that first sentence. Since all stars upon
between cow’s milk and colic. Option (A): this formation contain lithium, and all stars except the
answer supports the conclusion by showing that coolest brown dwarfs can destroy lithium
genetics in likely the cause of colic (hence not completely, any star that lacks helium is not one of
cow’s milk). Option (B): this answer also supports the coolest brown dwarfs. The argument doesn't
the argument by showing that infancy itself could be make sense, because stars do not necessarily stay
the cause of colic (hence not cow’s milk). Option the same over their histories. For instance, there is
(C): This answer also supports the question, as the no reason given to suppose that one of the hottest
vast majority of infants fed cow’s milk did not brown dwarfs couldn't destroy its lithium and then
develop symptoms of colic (hence cow’s milk is not cool down later, thus becoming one of the coolest
the cause). Option (D): this is the correct option. brown dwarfs, while lacking lithium. You are asked
Because the elimination of cow’s milk led to a on which assumption the argument depends, so
complete disappearance of colic, the conclusion you should focus on that unwarranted assumption.
that cow’s milk is unrelated to colic is weakened. Option (A): This is the correct option. The argument
Option (E): this answer also supports the must assume that those coolest brown dwarfs have
conclusion by suggesting that an infant’s digestive not, in the past, been hot enough to destroy their
system is the cause of the development of colic lithium. Option (B): Importantly, this incorrect choice
(hence not cow’s milk). treats the first sentence as if it plays a part in the
13. (D); the main issue is addressed in option (D). As per argument. Since the first sentence merely supplies
Yolanda harm caused by joy riding is the most extraneous information, it is not connected to the
dangerous one and more as compared to main line of reasoning, and you should not believe
intellectual property harm caused by computers. that any connection is necessary. Also, since the
This particular stand is argued by Arjun. Option (A) question makes conclusions about certainty,
highlights only 1 element of the situation. Option (B) assumptions about majorities may not be helpful,
again highlights same single element concerning and in any case this choice supports a conclusion
computer data. Option (C) draws comparison about most brown dwarfs, but the question only
between general terms and misses specific details made a conclusion about the coolest brown dwarfs.
as per given question. Option (E) is extreme option Option (C): Since the question makes no
hence eliminated. conclusion about the destruction of helium, this
Head Office: 127, Zone II, MP Nagar, Bhopal |+91-9111555433| www.legaledge.in Page 4 of 26
Marathon - I
choice is irrelevant and incorrect. You should not can be concluded that the mixture is not effective.
have assumed that since helium is the result of the Remaining all the options is supporting the
destruction of lithium, the question claims that the argument.
helium persists. A result can in turn be destroyed, 21. (A); The cold sufferer begins with a hypothesis that the
and the actual fact that helium requires a pretty mixture were effective. If that hypothesis were true,
high incineration temperature is irrelevant. Option everyone with a cold would be using the mixture.
(D): The question states that stars are born Because not everyone does, it cannot be effective.
containing substantial amounts of lithium, and it is Observant test-takers will notice the conditional
in no way necessary to assume that they contain reasoning that underlies this argument:
roughly equal percentages. You should not assume Premise (1): ..... Effective → Everyone uses
that detection capability is relevant to the argument, Premise (2): ..... Everyone uses
and this choice is wrong. You need to differentiate Conclusion: ..... Effective
between the structure of an argument and the Essentially, the cold sufferer establishes that her
building blocks and results of the argument. When hypothesis (about the effectiveness of the herbal
criticizing an argument or finding assumptions, you mixture) is false by virtue of showing that it would, if
need to be interested in whether the premises true, require something that is false (everyone
presented a complete picture, and whether the using the mixture). Option (A) is the closest answer
premises definitely lead to the conclusion. You are to this prephrase and is therefore correct.
not actually interested in whether the premises and Option (A) This is the correct option. It is important
conclusion are correct. Option (E): Since the to understand the abstract language used in this
comparison between brown and red dwarfs was answer in terms of the actual substantive elements
utterly irrelevant to the main line of reasoning, this of the argument:
incorrect choice cannot represent a necessary Claim = mixture is effective
assumption. Consequence = everyone using it
17. (D); In this question we have to analyze the reasoning False consequence = not everyone using it
mentioned in each option and compare it with the False claim = mixture not effective
given condition. The reasoning should be Option (B) The cold sufferer does not accept a
analogous, drawing parallelism and must be flawed claim; she rejects one. This option is incorrect.
as per the given condition. The main reasoning is – Option (C) The conditions required to establish the
when a company loses lawsuit, its stock falls. Since truth of the claim that the mixture is effective are
Cotoy was involved in a lawsuit, same happened actually not met, since not everyone is using the
with it. Here, the major flaw is there can be some mixture. This option describes exactly the opposite
another reason for fall in value of stocks not the of what the cold sufferer does and is therefore
mentioned lawsuit can be held responsible for this incorrect.
every time. Option (A) cannot be the answer as Option (D) The author does not base a
treatment of customers discourteously leads to generalization on a representative group of
customers’ interest in some other shop. This is a instances: she only hypothesizes what would
criterion made for the employees, mandatory to happen if the mixture were effective. This option is
follow and drawing conclusion on the basis of this incorrect. Option (E) The cold sufferer does not
criterion. This is not analogous to the above given suspect the mixture would make recovery more
condition. Option (B) presents a statement and difficult. This option is incorrect.
conclusion drawn on the basis of this statement. 22. (E); The argument concludes that holding criminals
This is not similar to reasoning mentioned above. responsible for their crimes is equivalent to failing to
Option (C) again draws a mandatory conclusion on recognize that for all people actions are the product
the basis of some cause and effect relationship. of an environment that forged the agent's character.
This is not similar to our question situation. Option The argument asserts that the law-abiding majority
(D) is the correct answer as it is flawed argument. forges the environment, so it is the law-abiding
Because receiving $10,000 from college doesn’t people who are responsible for crime.
compulsorily proves that Eula has won the The reasoning in the argument is absurd, since the
performance scholarship. This amount can be for argument is self-contradictory. If people are the
something else, like refund etc. Option(E) can be product of their environment, then the law-abiding
negated directly as it is presenting an contradictory majority is every bit as much such a product as is
situation. the criminal element. At that point, if the criminal
18. (C); option (A) contradicts with the given situation. element is not responsible by reason of
Option(B) introduces a new topic of Carbon-di- environment, why should the law-abiding element
oxide which is not helpful in answering the be? The argument is based on contradictory
question. Option (C) is the correct answer as it assumptions. Since you are asked to describe the
provides possibility of insect population growth flaw, you should focus on the argument's absurd,
which can apparently solve given paradox. Option contradictory nature. Option (A): The argument
(D) and (E) are out of the context questions. does not exploit any ambiguity in the term
19. (D); since the only language classes being taught next "environment," so this choice is wrong.
term are advanced ones hence Professor Alban will "Environment" means "surroundings" and the term
not teach introductory level classes. We have to was very clearly used to mean that. Option (B): The
match the pattern of reasoning in the given options. argument very generally does distinguish between
Only option (D) is the appropriate choice. acceptable and unacceptable actions in that it
20. (C); only option (C) cannot be a valid assumption by the acknowledges the existence of criminal and law-
author. The main argument is since there are many abiding persons, so this choice is wrong. Option
people with cold and do not use mixture, hence it (C): Since the argument did not concern the
Head Office: 127, Zone II, MP Nagar, Bhopal |+91-9111555433| www.legaledge.in Page 5 of 26
Marathon - I
methods by which one becomes a criminal, but delaying or preventing the onset of pain. Notice the
instead where the blame lies for those methods, degree of likelihood expressed by the correct
this choice is not related to the main line of option: "there is a strategy that can be effective in
reasoning, and is incorrect. Flaw responses not delaying" the onset of pain. Proving that a certain
related to the main line of reasoning are in general outcome is possible is a lot easier than proving that
incorrect, even if you think the flaw may have this outcome is certain. Extreme language is rarely
existed. Furthermore, this choice is actually false, the hallmark of a good must Be True answer.
as the question does no such thing. Option (D): The correct option is (B)
Since the question concerns only an abstracted 24. (E); The question consists of information about coal in
discussion of "criminals" and "law-abiding majority" Country Q. First, we find out that at the end of each
without any clear specifics or comparisons to a year, a tally is made of Country Q's available coal
general population, you should not assume that an supplies, which are defined as the amount of coal
overgeneralization has occurred. This choice is that has been mined but not consumed. It is very
incorrect. Option (E): This is the correct option. important that you interpret that statement correctly.
Refusing to blame criminals for their actions on the The tally does NOT refer to the amount mined that
basis that all actions stem from environment, and year minus the amount consumed that year. What
then asserting blame onto the law-abiding, even the tally refers to is the amount of coal, total, that
though the discussion of criminals would more the country has left over after consumption. That
consistently commit the question to the proposition means that the tally incorporates the "roll-over" from
that no-one can be blamed, is an internal the previous years. Most people who have trouble
contradiction. with this question have misread that sentence.
23. (E); chronic back pain is usually caused by damage to Second, we learn that the tally in Country Q is lower
the spinal disk that occurs years before such pain in 1991 than it was in 1990, and that Country Q
develops. According to the last sentence of the does not get its coal from anywhere else, or export
question, a contributing factor to the onset of any coal. If you misunderstood the first statement, it
chronic back pain among people over the age of 30 would be impossible to infer anything definite from
is the deterioration of the abdominal muscles the question. However, once you understand that
caused by insufficient exercise. Option (A): Just the tally is a running tally, we know that if it has
because one person out of five has a damaged decreased from 1990 to 1991, it must be true that
spinal disk guarantees neither that this person will Country Q is cutting into its reserves. In 1991, it
develop chronic back pain, nor that others will avoid consumed more than it produced. Option (A): Once
it. It is entirely possible some spinal disk injuries do you understand that the year-to-year comparison is
not lead to chronic back pain; likewise, it is also irrelevant, you can eliminate this choice easily. If
possible that other conditions such as obesity or you misread the first sentence, you should still have
bad posture also cause chronic back pain. Option eliminated this choice. Even if you thought that the
(B): While regular exercise may be necessary to tally only took into account coal mined and
delay or forestall the onset of chronic back pain consumed in a specific year, it would be impossible
among those with injured spinal disks, there is no to say why the tally was higher in one year,
evidence that exercising is sufficient to prevent because either production or consumption could
back pain. Furthermore, causes for chronic back account for the change. Option (B): This is the
pain other than injured spinal disks might not be correct option. A correct reading of the question
manageable by exercise at all. This option is indicates that in the year 1991, Country Q cut into
incorrect. Option (C): While in most cases the disk its reserves, so that means more coal was
will have been damaged years before the pain consumed than was produced in 1991. Option (C):
develops, it is entirely possible that in some cases This option is wrong for the same reason that
chronic pain develops a lot sooner. Furthermore, option (A) is wrong. You should realize that under
even if chronic pain always took years to develop, the incorrect reading of the question, either of these
option (C) only talks about "mild and fleeting" back choices could explain the situation, but neither of
pain. It is quite possible that many patients them must Be True. Option (D): This choice is
experience such pain at the time their spinal disks wrong, because you should not assume that
are injured, since "mild and fleeting" back pain is production necessarily remained constant.
not the same as "chronic" pain. Option (D): While Remember, under any reading, it is the relationship
doctors may be able to predict which spinal disk of production to consumption that changes, and we
patients will develop back pain (e.g. those who fail are not certain individually how the two have
to exercise), there is no way to make a similar changed. For example, consumption could have
prediction if the group consists of ordinary people decreased, while production decreased more. Or,
without chronic back pain. At best, doctors may be production could increase, while consumption
able to diagnose the 30+ year olds who have increases more. And there are more options still, so
herniated or degenerated spinal disks but show no this choice is no good for a must be true question.
symptoms (usually, one in five will fall into that Option (E): There is absolutely nothing to indicate
category). However, since injured spinal disks may we ought to compare halves of years, and this
not be the singular cause of chronic back pain, it is choice is incorrect.
impossible to estimate who else will ultimately 25. (D); Tom concludes that since employers complain that
develop back pain. Option (E): This is the correct high school does not currently give graduates the
option. If insufficient exercise is a contributing factor vocational skills necessary for full-time
to the onset of chronic back pain among people employment, high school students should have to
with injured spinal disks, it follows that exercising work part time jobs so that they will acquire skills for
represents a strategy that can be effective in the job market. Mary brings up the practical
Head Office: 127, Zone II, MP Nagar, Bhopal |+91-9111555433| www.legaledge.in Page 6 of 26
Marathon - I
consideration that jobs are already under-available, argument by showing that disrespectful
and simply requiring students to get jobs will not comedians are successful. The statement as
create jobs. You are asked to identify the actually written in the option tends to weaken,
relationship of Mary's response to Tom's argument, not strengthen, the author's argument. Option
so you must focus on the fact that she attacks his
plan by bringing up an important consideration that
(D): This is the best correct option. Try
may make his plan unworkable. Option (A) Mary assuming the logical negation of this option:
does not analyze any results of Mark's plan; she What if people who value an ideal especially
simply suggests that the plan is not even possible. highly do always succeed in living up to their
Option (B) Mary might argue that Tom's suggested ideal? In this case, there would be no "failings"
method of reversing the trend of producing for comedians to exaggerate, and thus there
vocationally unskilled graduates is unworkable, but would be no basis for successful comedy, as
that does not mean that Mary thinks that there is no described by the author. So the statement in
way to reverse the trend, so this choice is wrong. If this option is necessary to make the argument
you assumed that "trend" referred to the availability
of part-time employment, you still should have
work. Option (E): The question made no
eliminated this choice. There is no reason to reference to dynamic changes in popularity of
believe that Tom has even considered the comedians over time.
relevance of the trend in job availability, let alone 2. (C); (A) Offers in support of its conclusion pieces
that he made a mistaken evaluation of the trend. of evidence that are mutually contradictory :
Option (C) Mary's new information is inconsistent We are not told the two things are
with the idea that Tom's plan would be possible to contradictory. (B) overlooks the possibility that
apply; however, since Tom never states that his a person can practice law without being a
plan would be practical, Mary's new information is member of a law firm : No, it is actually saying
not an attack on Tom's explicit (stated) premises.
Option (D): This is the correct option. Mary presents
since that person is not practicing that thing,
the consideration of part-time job availability, which so he is not the member, This point is the
undercuts Tom's assumption that his plan is other way round. (C) Concludes that someone
workable. You might not consider it clear that Tom is not a member of a group on the grounds
makes such an assumption, but it is almost certain that that person does not have a characteristic
that someone arguing from Tom's position would that the group as a whole has : Yes, so this is
believe that any requirements put upon high-school what the author is saying. Since that person
students for their own good are requirements with doesn't have those characteristics, he cannot
which students can comply, so you should grant be the member of that group. Option (D) and
some leeway. Option (E) Mary offers no solutions,
so this choice is wrong.
(E) are irrelevant.
(Section-II) 3. (B); Premises: In the western part of the country
where triple trailers are allowed on some
1. (D); the critic highlights the tension between
highways, these vehicles have a smaller rate
people's high value on respect for others, and
of road accident fatalities per mile of travel
their appreciation for disrespectful comedians.
than other commercial vehicles. Intermediate
The critic explains this tension by saying that
conclusion: Conclusion 1 : Clearly, triple
disrespectful comedians highlight and
trailers are safer than other commercial
exaggerate regular people's failure to exercise
vehicles. Conclusion 2: Hence, opponents of
respect for others, and this portrayal makes
allowing triple trailer trucks are wrong in
people laugh (thus producing "successful
claiming that these trucks are more dangerous
comedy."). Option (A): This option would
than other commercial vehicles. The first thing
greatly weaken the author's argument, by
I was thinking about is whether those triple
showing that the fans of disrespectful
trailer trucks are (a) driving as much as other
comedians are not the population (people who
commercial vehicles, (b) whether there are
place a high value on respect for others)
significantly less triple trailer trucks on the
whom the author speculates about. This
road, which could bring down the average
cannot be the correct option. Option (B): The
fatalities, and (c) whether there are less
author merely said that exaggeration of
people on the roads in the west and therefore
disrespect "often" forms the basis of
less of a chance of crashing there. Lets jump
successful comedy. The author never said that
into the options: (A) is wrong because it is
such exaggeration is the only basis of
entirely out of scope. The amount of trailers it
successful comedy. So if other bases for
takes to haul as much weight as a single
successful comedy exist, this would not
trailer can is simply irrelevant. (C) is wrong
undermine the author's argument.Option (C):
because it doesn't matter what the opponents
Try assuming the logical negation of this
of another problem thought about that problem
option: What if few people disapprove of the
- it certainly doesn't tell us anything relevant to
portrayal of disrespect in comedy acts? This
our argument. (D) is wrong because it talks
would not disprove the author's argument;
about what is needed to drive the trailers,
indeed, it would strengthen the author's
which doesn't tell us about the safety of those
Head Office: 127, Zone II, MP Nagar, Bhopal |+91-9111555433| www.legaledge.in Page 7 of 26
Marathon - I
trailers relative to other commercial vehicles. these choices will resolve the paradox, but
Get rid of it. (E) is wrong because it's simply one choice will not. Option (A): The new light
an unnecessary comparison. It doesn't hurt bulb generates unappealing light. This
the argument by telling us how many explains why the new light bulb might be
accidents happened in one year compared to unsuccessful with customers. This option does
the other two years. Finally, (B) is our correct resolve the paradox, so it cannot be the
option because this hit nicely with our correct option. Option (B): This option explains
prephrase (C), which is that the reason is exactly why people might prefer the old,
accidents per mile in the west are lower is inexpensive bulb over the new, durable, but
because there is less traffic and therefore less expensive bulb. This option does resolve the
of a chance to get into an accident. It could paradox, so it cannot be the correct option.
very well be the case where the triple trailer Option (C): If the new light bulb runs up more
trucks are prohibited in the east because there electricity cost than the old bulb (or at least if
is more traffic and therefore more of a chance competitors are advertising this claim and
to get into an accident with these trucks. This customers believe it) then this might erase the
sheds a ton of doubt on the support for the new bulb's advantage in lifespan-per-dollar-
original argument's conclusion because it spent. This option does resolve the paradox,
shows that the support (the rate of accidents is so it cannot be the correct option. Option (D):
lower in the west) is actually due to something This is the correct option. The type of
else. It doesn't entirely destroy the argument, packaging of the bulb has little apparent
but it definitely gives us a damn good reason connection to the market success of the bulb;
to be suspicious of following through with the we are told nothing about packaging or
recommendation in the argument. quantities in the question. This option does not
4. (A) ; Whittaker tries to argue that there cannot be resolve the paradox, so this is the correct
a category of people that dropped out of option. Option (E): If a competitor is going to
medical school before their second year introduce a "super bulb" with an absolute
because if they dropped out they never advantage in cost and lifespan, this would
experience a second year. In response, trump the second bulb's advantage in lifespan.
Hudson follows this same logic to show that The second bulb will lose in the market to the
there is no category of people who will not third bulb. This option does resolve the
become rich because if you die before paradox, so it cannot be the correct option.
becoming rich, you never experience it. Option 6. (C); the key to answering this question quickly and
(A): This is the correct option. Hudson uses efficiently is to focus solely on those facts that
the analogy of becoming rich to show that if he can be put together to form a logically valid
uses the same logic as Whittaker, it leads to a inference. For instance, you should notice the
preposterous conclusion. Option (B):Hudson strong element of conditionality between full
does not cite anything specific that directly flooding and implosion: any sinking ship that is
counters Whittaker’s claim about second year not fully flooded will implode:
medical students. Instead he attacks it through Fully flooded :arrow: Implode
analogy. Option (C): Whittaker does not use According to the last sentence in the question,
necessary/sufficient reasoning in his the Rienzi apparently did not implode. By the
statement, therefore Hudson cannot point out contrapositive of the above-mentioned
such a mistake. Option (D): Whittaker makes a statement, the Rienzi must have been fully
simple statement without ever acting as if it flooded when it sank to the bottom of the
were false. Option (E): Hudson never claims ocean. While the author observes that full
that Whittaker’s use of medical school flooding can be achieved by sabotage, the
students is extreme and unrepresentative. relationship between the two is not conditional
5. (D); this question presents an apparent paradox. If but causal (the author only states that full
the new light bulb costs three times the price flooding "can be achieved by sabotage," not
of a conventional light bulb but its lifespan is that it necessarily "must" be achieved by
ten times as long, then it might seem that the sabotage). Therefore, we can only conclude
new bulb has a much greater lifespan-per- that sabotage was a likely cause for the full
dollar-spent. (A mathematical demonstration: flooding, a possibility that is also supported by
Say the old bulb costs $1 and lasts for 1000 the first sentence in the question. If sabotage
hours. Say the new bulb costs $3 and lasts for was not the cause, however, water must have
10,000 hours. The old bulb gives you 1000 flooded the ship unusually fast, since under
hours per dollar spent; the new bulb gives you normal circumstances it does not enter the
3333 hours per dollar spent.)Despite the ship quickly enough to fully flood it before the
apparent advantage, why would the new bulbs ship sinks to the bottom of the ocean. Option
perform poorly in the market? Look carefully at (A): It is entirely possible that the Rienzi sunk
the wording of the question stem. Four of by impact, which caused the water to flood the
Head Office: 127, Zone II, MP Nagar, Bhopal |+91-9111555433| www.legaledge.in Page 8 of 26
Marathon - I
ship unusually fast. How the ship was probability of the unvaccinated dogs
constructed is irrelevant and not inferable from contracting the disease.
the evidence presented in the question. Option 8. (A); the argument in this question is basically that
(B): This option is directly disproven by the if a mental disorder results from a chemical
information in the question, since any ship that imbalance, it can only be effectively treated
sinks deep into the ocean floor when not fully with medication that will reduce or correct the
flooded will implode, and the Rienzi did not chemical imbalance. The unstated
implode. Therefore, the Rienzi must have presupposition here is that other forms of
been fully flooded when it reached the ocean treatment (psychotherapy, say) cannot correct
floor. Option (C): This is the correct option. the chemical imbalance. In order to make the
Indeed, if sabotage was not the reason why argument work, we need to find an option that
the Rienzi was fully flooded and water cannot provides this missing assumption. Option (A):
enter the ship quickly enough, water must This is the correct option. Assume the logical
have flooded into it unusually fast. See opposite of this statement: What if
discussion above. psychotherapy can produce effective
Option (D): There is no reason to suspect that reduction/correction of chemical imbalances
had the Rienzi sunk more slowly, it would not that cause mental disorders? If this were true,
have imploded. The question contains no then the author's argument fails — medication
evidence to suggest that the speed at which would not be the only way to treat chemically-
ships sink has any effect on whether they caused disorders. So by the Assumption
implode or not. Furthermore, even if a slower Negation test, this statement is necessary for
sinking ship has a higher chance of being fully the argument to work. Option (B): The
flooded when it reaches the bottom of the argument makes no reference to the speed of
ocean, such a ship can still implode. To results; it discusses only the effectiveness of
conclude otherwise would be to rely on the treatment. So this option is irrelevant. Option
Mistaken Negation of the conditional (C): We do not need to assume that most
relationship between flooding and imploding. mental disorders have non-chemical causes.
Option (E): How the ship was built is Even if most mental disorders do have
immaterial to the information provided in the chemical causes, the argument can still work
question: the fact remains that any ship that — we would just be talking about a very large
sinks deep into the ocean when not fully number of disorders. Option (D): Assume the
flooded will implode. This option is incorrect. logical opposite of this statement: What if
7. (E); the argument is talking and gives information psychotherapy were sometimes more effective
about the following things: than medication in treating patients with
1. One out of every 50 dogs dies because of mental disorders? This deals generally with all
the disease mental disorders of various causes. This
2. Vaccine is a virtual 100% cure for the would not undermine the specific argument
disease about medication being the only effective
3. Vaccine has a death rate as well i.e. one treatment for chemically-caused mental
death per 5000 vaccines disorders. Thus, by the Assumption Negation
4. It is safer for a dog to receive a vaccine test, this option is not necessary for the
than not to receive it argument.
So as you can see (from above) that the Option (E):"Effective treatment" was not
argument gives a lot of info about vaccine and defined in the question. Does effective
also that one out of 50 dogs die if they are not treatment imply merely a reduction of
vaccinated. In order to evaluate the argument symptoms, or a deep correction of the
you would need to know - what is the underlying cause? This is not made clear. For
likelihood that an unvaccinated dog will this reason alone, we could reject this option.
contract the disease. Additionally, try the Assumption Negation test:
Scenario 1 - Let's say 1 in 10000 dogs Ask yourself, what if psychotherapy has an
catches this disease, then vaccinating all the effect on mental disorders beyond just
dogs is by no means a practical solution reduction of symptoms? Just like option (D),
because 1 in 5000 dogs can die after this is a very general statement about all
vaccination. mental disorders, and it does not defeat the
Scenario 2 - Let's say 1 in 10 dogs contract specific claim about medication being the only
that disease and you already know that 1 out effective treatment for chemically-caused
of 50 dogs will die. Therefore the dogs should mental disorders.
be vaccinated. 9. (B); due to the relatively complex structure of the
Therefore in order to evaluate the argument curator's argument, it is important to
you need to know the likelihood, percentage or understand it fully before proceeding to
answer the question:
Head Office: 127, Zone II, MP Nagar, Bhopal |+91-9111555433| www.legaledge.in Page 9 of 26
Marathon - I
Premise (1): ..... Reliable tests show that the cloak in Veronese's painting was not originally
red pigment was applied after the painting had painted red by the artist, and therefore the
been completed proposed restoration to the green underneath
Premise (2): ..... The red paint was not mixed is justified. The critic, in response, has
in Veronese's workshop provided evidence that the cloak was likely red
Sub. Conclusion: ..... An artist other than at the time of Veronese's death. To get to a
Veronese tampered with his painting after its prephrased answer here, we must combine
completion our knowledge that the proposed restoration
Main Conclusion: ..... The decision to restore will change the cloak from red to green, with
the cloak of the central figure in Veronese's the art critic's argument that Veronese
painting from its originally painted to cloak red.
present red to the green found underneath is Option (A): While this answer may be true, the
fully justified critic's statement does not provide strong
As is often the case with Method of support for it. All we know is that a copy was
Reasoning/Argument Part questions, it is made and the cloak was red. We have no
crucial to distinguish between the "subsidiary" information about the quality of the copy or
conclusion and the "main" conclusion of the whether it would be distinguishable from the
argument, particularly since the test makers original. Option (B): The curator's claim relies
will often try to trick you by prefacing the on technology, not the critic's. Since the critic
subsidiary conclusion with a typical conclusion provides no information, insight, or opinion
indicator ("hence") while providing no regarding the use of technology, this cannot
conclusion indicators for the main conclusion. be the correct option. Option (C): This is the
This tactic underscores the importance of correct option. The information provided by the
understanding the function of each statement. art critic tells us that the cloak was likely red at
Compare the two contender conclusions and the time of Veronese's death. Therefore, if the
check to see which one answers the question proposed restoration is going to change the
"why?" — whichever one does, this will be the cloak from red to green, it follows that the
subsidiary conclusion of the argument. In this restoration would not restore the painting to
instance, the decision to restore the cloak is the appearance it had at the end of
explained by the observation that someone Veronese's lifetime. Option (D): The argument
must have tampered with the painting. made by the art critic provides no information
Therefore, tampering with the painting is not sufficient to make a value judgment on
the main conclusion of the argument, even whether or not an artist's work is compromised
though two separate premises are used in its when that work is tampered with by later
support. By answering the question "why is the artists. Option (E): This one is tricky because it
decision to restore the color of the cloak seems to match with what the art critic is
justified?", the statement that someone else implying. However, if you look at the art critic's
must have tampered with Veronese's painting statement more closely, you will see that she
provides a premise for the main conclusion. is solely making a statement on what color the
Because this claim is both supported by the cloak was at the time of Veronese's death.
initial two premises and in turn supports There is no information that can be derived
another statement, it is a subsidiary from the art critic's statement regarding
conclusion of the argument. Option (B) is Veronese's original intent.
therefore correct. Option (A) As discussed 11. (D); the premises in the question establish the
above, the main conclusion is the first following logical relationship: "If there are
sentence of the question and not the last one, holidays or vacation, John works less than five
despite the misleading conclusion indicator days in a week. If neither holidays nor
("hence") provided here. This option is vacation are present, John works five days per
incorrect. Option (B): This the correct option. week." We are told that last week there were
Option (C) This statement does not clarify a neither holidays nor vacation. The obvious
key term. This option is incorrect. Option (D) conclusion is that John worked five days last
There is no general position in the curator's week. We know that Friday he worked as a
argument that needs defending. This option is blacksmith. But how can we be sure that his
incorrect. Option (E) While the main point is in other days of work were Monday through
the first sentence, the last one does not Thursday? Why are we so quick to dismiss the
reiterate it. Rather, it provides a rationale for it, possibility that he worked on Saturday or
by explaining why the decision to restore the Sunday? This is the missing assumption.
color of the cloak is justified. This option is Option (A): Apply the Assumption Negation
incorrect. test. Even if John sometimes takes a vacation
10. (C); the curator in the question has provided of more than one week in length, it doesn't
evidence supporting a conclusion that the matter for the argument: we know that John
Head Office: 127, Zone II, MP Nagar, Bhopal |+91-9111555433| www.legaledge.in Page 10 of 26
Marathon - I
had no vacation last week, so any possible that the obstruction of sunlight makes
consideration of vacation is irrelevant. Option it impossible for any seedling to succeed in a
(B): The argument is not really concerned with stand of white pines, it does not follow that
the amount of time that John worked on other species of trees would colonize and
individual days. The argument is only making replace the stand of white pines. This choice is
a claim about the identity of the individual days unsupported, somewhat contrary, and
when John indeed worked. Option (C): Like incorrect. Option (E): The question does not
choice (A), this statement is irrelevant. We establish that white pines grow at a fairly
know that neither vacation nor holiday invariable rate. Furthermore, the question
occurred last week, so we have no reason to tends to suggest that the trees are all of very
care about vacation or holiday. similar ages if they are to survive together,
Option (D): This is the correct option. Without which tends not to support the idea that the
the statement in this option, it becomes very height differences should be attributed only to
possible that John worked on days besides age.
Monday through Thursday. Option (E): Apply 14. (C); Option (C) says that it concludes one thing
the Assumption Negation test: What if there (let's call this Q) was caused by another (call
were some (at least one) days last week in this Z), so: The argument apparently claims
which John worked both jobs? Would this that Z causes Q. If Q was caused by Z, this
destroy the author's argument? No: it is matching makes sense. Let's continue:
conceivable that John could have worked his ...although the evidence given is consistent
insurance job on Monday, Tuesday, with the first thing's (that's Q, as it was the first
Wednesday, Thursday; and that John worked thing listed in this option) having caused the
the blacksmith job on Thursday and Friday. second (that's Z then).So this is saying that
This story is perfectly consistent with the the evidence is consistent with Q causing Z.
premises of the question. This, then, is describing a reverse cause and
12. (B); only option (B) justifies the judgement about effect flaw. If the argument claims that Z
George and Carla. Other options are not causes Q, then we need to look at what the
appropriate as per the question situation. argument claims to identify Z and Q with the
13. (A);The question consists of premises, but drives phenomena in the argument. The argument
toward a fill-in-the-blank conclusion. You must concludes that owing a laptop led to a higher-
infer the main point of the argument. The paying job, so the argument says: owning a
premises establish that mature white pines laptop causes a higher-paying job and Z is
obstruct sunlight well enough that such pines "owning a laptop" while Q is "having a higher-
cannot regenerate in their own shade, and you paying job". Because the option was phrased
need to decide what is likely in a stand of in the passive voice, the "first thing" listed is
white pines in a dense forest. You are actually the claimed effect in the argument, not
supposed to pick up on the theme of sunlight- the cause.
obstruction, and realize that as a stand of 15. (B); This question is easier to understand if you
white pines becomes thicker, it becomes less pay attention to the formal logic content:
likely that new white pines will grow. That Premises: Rhonda goes to movie ..... :arrow:
would suggest that a dense stand of white ..... Paul goes to concert
pines consists of trees of fairly similar ages. Paul goes to concert ..... :arrow: ..... Ted goes
Option (A): This is the correct option. At the to concert
very least, within the stand the pines should Ted refuses to go to the concert
not differ drastically in age. According to the Conclusion:
question, younger white pines would not Ted goes to the concert ..... :arrow: .....
survive if they had to grow under the cover of Rhonda goes to the movie
older white pines, so you would expect that a The reasoning is logically valid, and involves
close grouping of white pines consists of trees the contrapositive of the additive inference.
of fairly similar ages. Option (B): The area Option (A): This option does not link the first
would not have to be cleared of all trees, just element to the last, so is dissimilar.
enough trees that the white pine saplings Furthermore, this choice involves a Mistaken
could get enough light to grow. White pines Reversal of the premise:
cannot regenerate in their own shade, but Janice visits ..... :arrow: ..... Janice finds baby-
might be able to regenerate in a lesser amount sitter so this choice is logically flawed, and
of shade. Option (C): The question suggests incorrect.
that older white pines block the sunlight to Option (B): This is the correct option. It can be
seedlings, but there is no reason to assume diagrammed as:
that older white pines would deprive each Gary does laundry ..... :arrow: ..... Peter has to
other of sunlight enough that some of the older work ..... :arrow: ..... Cathy is ill
trees would die. Option (D): Since it is quite Cathy is ill ..... :arrow: ..... Gary does laundry
Head Office: 127, Zone II, MP Nagar, Bhopal |+91-9111555433| www.legaledge.in Page 11 of 26
Marathon - I
This illustrates that this response involves a only one government scientist who receives
correct additive inverse and uses the such a premium, and perhaps he is receiving
contrapositive, just as was done in the the premium for entirely unrelated reasons
question. (maybe he's dating the boss). The author's
Option (C): Since the first premise contains fundamental argument could still hold in this
two sufficient conditions, this response fails case. Thus, the statement in this option is not
the Premise Test, and is incorrect. You should absolutely necessary. Option (C): The number
immediately eliminate this choice. of scientists currently employed in government
Option (D): Since the first premise contains a jobs vs. private sector jobs is irrelevant to the
choice of necessary elements, and the issue of the incentives for scientists to switch
question did not, this response fails the jobs. Option (D): This is the correct option.
Premise Test. Eliminate this choice Apply the Assumption Negation test: What if
immediately. government scientists do get unusually good
Option (E): This response does not link the working conditions or fringe benefits that more
first and last elements. Also, this option than compensate for low salaries? This would
contains a Mistaken Reversal of destroy the argument; in this case, clearly self-
Mark goes to the museum ..... :arrow: ..... interest would not push the scientists toward
Postponed appointments so this choice private-sector jobs. Thus, the statement in this
contains fallacious reasoning, and is incorrect. option must be necessary for the argument.
16. (D); Premise: Scientists in the private sector can Option (E): Applying the Assumption Negation
earn 50 percent more than in a government test: What if private-sector scientists work
job requiring similar skills. Additional premise: shorter hours? This would actually support the
Government-employed scientists would have argument, not destroy it.
no problem finding private-sector jobs. 17. (E); This question provides that although efficient
Conclusion: The most-skilled government use of fossil fuels is in the interest of the
scientists will leave for the private sector, nation and the world, government standards
unless their sense of public duty trumps their are needed in order to make improvements.
self-interest. This argument implicitly assumes The objection: Energy use should be
that monetary compensation is the only factor determined by the market. The question is
entering self-interest. What if the government followed by another weaken question, so we
jobs are safer? What if the government jobs should look for the option which would cause
have better job security? What if the us to question the assertion that the market
government jobs have more reasonable should rule in such contexts. Option (A): There
hours? Any of these factors could undermine is no mention of implementation "all at once,"
the argument. Thus, to make the argument and no suggestion that either government
work, it is necessary that we dismiss all of standards or market forces might immediately
these possibilities. Notice the question stem: maximize efficiency. Since this has no effect
We are looking for a necessary, though on the argument that market forces should
perhaps not sufficient, assumption which will rule, this option is incorrect. Option (B):
fill in at least one gap in this argument. Option Whether or not certain products allow for easy
(A): Applying the Assumption Negation test: assessment of inefficiencies, this makes it no
What if government scientists are equally or more or less advisable to leave fossil fuel
more likely to receive recognition and kudos decisions to the market vs. the government,
for their work? This would undermine the this option does not weaken and is therefore
argument, as it suggests an additional reason incorrect. Option (C): The difficulty of
why public-sector work might be more implementation does not weaken the author's
attractive. This option looks like an attractive argument about market forces, so this option
candidate. However, note that we do not is incorrect. Option (D): This option would
completely destroy the argument by applying strengthen the argument that market forces
the Assumption Negation test. Private-sector would naturally find efficiency in the use of
jobs could still be better in terms of self- fossil fuels. Option (E): This is the correct
interest, even if they do not win on the option. If the purchase of such products is
recognition factor. Option (B): What if some often made by someone other than the end
government research scientists earn more user, then market forces would not be as likely
than the highest-paid private-sector scientists? to maximize efficiency in every case.
At first glance, this seems to undermine the Landlords might, for example, buy an old,
argument: perhaps the government is giving a cheap, inefficient dishwasher to save money,
salary premium for scientists at the very top of knowing that tenants will be left with the higher
the skills ladder, and thus such scientists will bills.
not leave. However, this story would not 18. (A); Context: Some historians claim that the
entirely destroy the question. Perhaps there is people who built a ring of stones thousands of
Head Office: 127, Zone II, MP Nagar, Bhopal |+91-9111555433| www.legaledge.in Page 12 of 26
Marathon - I
years ago in Britain were knowledgeable which is an opinion itself. This is why this is a
about celestial events. The ground for this tricky option. You see the word "opinion" and
claim is that two of the stones determine a line you jump on it. But we need to break down
pointing directly to the position of the sun at what the statements are actually saying before
sunrise at the spring equinox. we can make a judgement call about it being
Premise: There are many stones in the ring, the correct answer. In this case, this is 100%
however, wrong. (E) Dobson’s drawing the conclusion
Conclusion: so the chance that one pair will relies on interpreting a key term in two
point in a celestially significant direction is different ways -- No terms conflict. If we used
large. Therefore, the people who built the ring "knowledgeable" or "rings" in different ways
were not knowledgeable about celestial this would be the case, but we do not do this.
events. So we are told about a theory where 19. (D); the argument starts off with an estimated
certain objects helped people, potentially, a delivery time for mail that is correctly
while ago spot items in the sky. But our addressed and ends with an estimated
argument is that there are a bunch of these delivery time for "all" mail. Option (A) cannot
items, so the fact that there are so many be concluded with the info in the argument.
probably means it was an accident. Hmm. So We only know about correctly addressed mail
we are saying that, in our opinion, that if you and incorrectly addressed mail. It could be that
something could be a coincidence, not the delay of 3 days or more is from the
purposeful, that that destroys the argument. incorrectly addressed mail. Option(B) takes
But this doesn't destroy our argument because the conclusion too far. Option (C )- corollary is
it is simply our opinion(A) The failure of cited not true. Correctly addressed mail comes
evidence to establish a statement is taken as within 2 days but not all mail that arrives within
evidence that that statement is false -- Let's 2 days are correctly addressed mail. Option
break this sentence down: The failure of cited (E) is incorrect as it contradicts the last
evidence (the rings) to establish a statement sentence of the argument. Option (D) is just
(that they point to sky items on purpose) is better compared to the other options. Since
taken as evidence that that statement is false correctly addressed mail takes 2 days or less,
(They are not experts). Hmm. Pretty good. We we know that the contributing factor for the
are told something is a fallacy because the overall 3 days or more are the incorrectly
evidence presented doesn't add up. But this is addressed and "damaged in transit" kind.
just one example. Can we conclude that there 20. (E); the argument is essentially that since Senator
weren't other structures? Further, can we see Armand maintains that the program could not
how the structure matches well with what the be successful, the figures cited in the report
actual argument did? We are told that the are not accurate.
author doesn't believe it because of ` That correctly utilizes the contra positive of
coincidence. No additional evidence is given. "Figures accurate → Program Successful."
He is just restating his opinion. So looking at All of the figures and the conditional are
the option, because we have no other designed to distract you from the real flaw: the
evidence and we disagree with it, we assume argument is based on the inappropriate appeal
that the statement is false. Perfect. (B) to the expertise of Senator Armand. It doesn't
Dobson’s conclusion logically contradicts seem likely that we should reject 5 years of
some of the evidence presented in support of study just because some "smart" person off
it -- Our premise matches our conclusion the top of her head decides that certain things
pretty well. This would be like saying I hate red are impossible. We'd rather see the numbers,
items. And all apples are red. Therefore, I love and decide for ourselves. Option (A): This
apples. Option (C) Statements that absolutely option does not reference an authority-- in
establish Dobson’s conclusion are treated as if fact, even though the father might be the
they merely give some support to that authority in some senses, the father's claim is
conclusion -- This is the opposite, in a way. rejected. What this choice does do is side with
This is saying that the question doesn't believe the person who claims to have an eyewitness
the arguer's statement. But it is the other way account, which is the opposite of what the
around. (D) Something that is merely a matter question does. Option (B): This option sides
of opinion is treated as if it were subject to with the child, not the authority, and weighs at
verification as a matter of fact -- Trap option. least some eyewitness claims more heavily
This is saying that the view about the rings is than other claims, so this choice is incorrect.
being treated as a matter of fact when it is Furthermore, this choice contains a different
opinion. It is established that this is fact and flaw. A failure to carry out a policy 100% does
we are arguing against it. The error is not not prove that the policy doesn't exist,
giving too much credit to the claim we are because policies define what people will
disproving. The issue is in our argument, attempt to do rather than what people are
Head Office: 127, Zone II, MP Nagar, Bhopal |+91-9111555433| www.legaledge.in Page 13 of 26
Marathon - I
successful at doing, so the conclusion is Werth cites. (C) is incorrect because Chang
unjustified. Option (C): Even though Amos doesn't address expertise, and Werth pretty
could simply be an extreme freak of nature, clearly cites the right experts. (D) is just way
this response involves the correct application out. Werth doesn't disallow any kind of
of expertise. Dr. Treviso has specific, studied evidence. (E) is incorrect because likelihood
expertise that would enable him to make his vs. possibility never comes up.
judgment, and accepting his claim is not at all 23. (D); This question begins with the author's
the same as accepting Senator Armand's, conclusion: garbage dumps do not harm
whose claim is not informed. Option (D): This wildlife. One way to tell that this is the central
option involves a Mistaken Reversal of its last belief is that the remainder of the information
premise, "Robert is right → Evelyn did not is classified as "evidence," meaning given to
listen to late news." The question did not support a view. So what evidence are we
involve a formal logic error, so this response is offered to conclude that garbage dumps do
wrong. Furthermore, this response does not not harm wildlife? The single example of
involve an inappropriate appeal to authority. baboons on the Masai-Mara reserve in Kenya
Option (E): This is the correct option. Just that use local garbage dumps as a food
because Lomas is an engineering expert does source. Members of this baboon troop mature
not mean he has the ability to judge which faster and have more offspring than do
athlete has a chance of winning. It makes no baboons on the reserve that do not feed on
sense to reject an eyewitness account in favor garbage.
of Lomas' "authority." This may seem at first reading like a
21. (D); the correct option is (D). Wirth clearly would somewhat convincing argument—after all, if
agree with option (A). Chang's opinion is more garbage-consuming baboons mature more
difficult to establish. Remember that Chang quickly and reproduce more readily than their
did say, "I do not dispute your evidence." This garbage-free neighbors, isn't it reasonable to
means that he agrees that no evidence has conclude that they're healthier?—but when we
been found to prove that a single gene is start to really scrutinize the assumptions made
responsible for predisposing people to manic- here problems quickly arise. First,
depression. Chang also states that comparisons are tricky. To make a truly
researchers have "found evidence that a set of legitimate sweeping comparison, like overall
several genes is involved," but this implicitly fitness, between two things, like baboon
indicates that they have only found evidence groups, you need to know a lot of information
of interaction, not actually identified the genes about them. They need to be of the same
themselves. Thus, Chang would be forced to type, for instance, otherwise what's the point
agree with option (A). Since both agree, option of an A/B measure? Second, the conclusion is
(A) must be incorrect. Option (B) is incorrect that "garbage dumps do not harm wildlife."
because both would probably disagree with That's an incredibly broad belief on three
the statement it contains. counts: dumps in general (rather than just
garbage as food), no harm whatsoever, and all
22. (A) ; we don't need to assume anything here. wildlife. For the first, we can't classify dumps
We're being asked to describe Chang's safe merely because it might be safe to eat
criticism of Werth's argument. What does their contents. Many, many other hazards
Chang think Werth is wrong about? Werth could arise from dumping garbage wholly
concludes that if it's true that there's no unrelated to its consumption. For the second,
"manic-depressive gene," it must be false that multiple measures of health, certainly more
anyone has a genetic predisposition to manic than just two superficial physical observations,
depression. Clearly, his assumption is that a would be needed to conclude something so
manic-depressive gene is the only thing that generalized as "no harm." And for the third,
could cause this type of genetic predisposition. can you really draw a conclusion about wildlife
Chang identifies this assumption indirectly and on the whole from some information about a
weakens Werth's argument by introducing a single troop of baboons?
specific alternative to his assumption: With that in mind, it seems clear that the
interactions among genes, rather than one implied causality between eating garbage and
single gene, cause manic depression. So improved fitness cannot be fully trusted, nor
Chang's issue with Werth is that he does can the jump from either eating garbage to
presuppose only one possibility (that a single general dump safety or from baboons to all
gene causes manic depression) when wildlife. So this argument is actually riddled
according to her, at least one other possibility with issues, making it ideal for a Weaken-
(interactions among multiple genes cause it). Except question stem. We'll have four
So (A) is correct. (B) is the opposite of what (incorrect) answers that undermine the
Chang says; she agrees with the evidence conclusion, and a single answer (correct) that
Head Office: 127, Zone II, MP Nagar, Bhopal |+91-9111555433| www.legaledge.in Page 14 of 26
Marathon - I
does not—it may help, or it may be irrelevant; the dump does indeed cause some harm to
it simply can't weaken. wildlife. It's another instance of the cause
Option (A): This is, in my opinion, the most likely tied to an effect that contradicts the
interesting of the wrong, weakening answers. author's claim, and thus weakens the
To many it seems irrelevant, but think about argument.
what it's telling you: if the two baboon groups 24. (B); Marianne is a professional chess player who
being compared to establish one group's hums, disrupting the other players. She is
superior fitness are in fact different species, ordered to stop humming or be disqualified,
then the comparison itself becomes but she argues that she is unaware of her
meaningless! Maybe the species that eats humming, so it must be involuntary and
garbage is supposed to mature faster and therefore should not be held responsible for it,
have more offspring. Noting these differences and concludes by means of protesting the
is only useful if they're unexpected or unusual, order. Hopefully, Marianne's chess play is less
but with two different species there's no way to dependent upon the distraction of her
tell if the maturation/reproduction "success" is opponents than is her apparent reasoning on
actually success, totally expected, or even the credulity of the officials. Marianne
less/lower than it should be. presumes that awareness and voluntary action
In other words, the possible effects of garbage are factors in responsibility, and you are asked
consumption can't be known at all if the to find the principle that supports her
comparison it's based on is irrelevant. argument. Option (A): This choice would
Option (B). This answer, along with the other merely provide for Marianne's opponents to
two wrong answers not yet discussed, return the favor, but would not justify
presents a direct attack on the health of the Marianne's assertion that she should not be
garbage-eating baboons: their life expectancy held accountable for her actions. This choice
is significantly lower than that of the other is wrong. Option (B): This is the correct
group. This clearly weakens, since it shows choice. If only voluntary actions are grounds
that there may well be a harm to eating for disqualification ("DQ→Voluntary Actions"),
garbage after all. In essence it shows the that means that involuntary actions should not
cause (eating garbage) possibly leading to an bring disqualification. That does not prove
opposite effect (diminished health). Marianne's argument, because it is possible
Option (C): In a near-copy of option (B), we that her lack of current awareness does not
have another negative health statistic for the prove that she is acting involuntarily, but it
garbage-eating troop: cholesterol levels does provide support for her presumption that
dangerously higher than those of the other whether an action is voluntary is a factor in
baboons mentioned. Once again, it's a cause accountability. Option (C): If a person should
leading to a different/conflicting effect, be held accountable for involuntary actions
severely weakening the argument. that serve a person's interest, that makes it
Option (D): This is the correct answer. Saying highly likely that Marianne's argument is
that a population of hyenas living near unconvincing. This choice would undermine
garbage landfills has doubled has absolutely Marianne, so this choice is wrong. Option (D):
no harmful effect on this argument, primarily The idea that normally involuntary behavior
because (1) we don't know anything about the should be presumed voluntary at a
hyenas' behavior (do they eat the garbage?), professional chess match serves to invalidate
(2) we don't know if the doubling population is Marianne's assertion that she should not be
normal, healthy growth or if it should have held responsible for her humming, so this
increased even more, and (3) we can't choice is wrong. With this principle, we might
assume any connection between that growth normally accept that random humming is
and the landfill itself. involuntary, but might not make that
If anything, one could possibly argue that this consideration in a chess match. Option (E):
answer strengthens the argument slightly—the This choice does not clearly attack Marianne,
garbage dump they live near clearly hasn't because you do not know that she is actually
reduced their population size, so maybe it is attempting to distract her opponents.
as harmless as the author believes—but However, it does not support her, either, since
regardless it should be clear that this answer it provides sufficient grounds for disqualifying
doesn't weaken the conclusion. Because it's people, but Marianne wants sufficient grounds
the exception here it is correct. for ruling out her humming as something for
Option (E): If you understood the negative which she can be disqualified.
effects of (B) and (C), (E) should present no 25. (D); Premise 1: Marianne is a professional chess
problems. If birth defects have doubled for the player who hums audibly while playing her
baboons on the reserve since the landfill matches, thereby distracting her opponents.
opened, then it increases the likelihood that
Head Office: 127, Zone II, MP Nagar, Bhopal |+91-9111555433| www.legaledge.in Page 15 of 26
Marathon - I
Premise 2:When ordered by chess officials to screen, so no assumptions about other systems are
cease humming or else be disqualified from required. This choice is wrong. Option (B): This
professional chess, Marianne protested the choice seeks to rule out the possibility that people
order. She protested saying that she is not who currently do not use Braille will be able to use
aware of her humming as she correlates the screens. Since the question does not concern
humming to be an involuntary action. anyone except current Braille users and is not
Conclusion : therefore she should not be held written to rule out other users, it does not matter
responsible for it. Not all of a person's whether other people will be able to use the
involuntary actions are actions of which that system, so this choice is irrelevant and incorrect.
person is unaware.This choice says some(not Option (C): This is the correct option. The argument
all) - which means some of the involuntary assumes that what people can sense by raised-dot
actions are done knowingly . But what if patterns they can also sense by heated patterns.
humming which is also an involuntary action Option (D): Since the discussion in the question
does not fall under the some category thereby simply concerns the viability of the new system of
strengthening Marianne 's argument that rendering Braille, it really does not matter whether
humming was left unnoticed by her? Hence C
there are other systems available. The point of the
is incorrect. But D clearly explains that
question was not that the new system is best; it was
humming is something that an individual can
only that the new system will work. Option (E): If it
notice easily and also can control , hence
is eventually possible to train people to use Braille
weakening her argument.
by training them on the thermal screen, then it is
26. (B); Since this is a resolve--except question, all
options but one will help to explain the very likely that the system can currently be a good
puzzling fact from the question. The correct substitute for raised dots. However, the question
option will not provide such a resolution. Here concerned people who already use Braille, not
the author tells us that despite a big drop in those who are to be trained in the future.
smoking, which has long been the main cause Furthermore, since even if the thermal screen is not
of home fires, there has not been a a good training device, it could be an acceptable
corresponding drop in the number of deaths substitute for people who already use Braille, this
caused by home fires. Option (A) tells us that choice does not represent a necessary assumption.
when people smoke in bed it doesn't generally 2. (B); In this question, the mayor states that the town has
lead to much damage. This would help to agreed to try to attract more residents, then makes
explain why, even with a significant decline in the claim that the best strategy for doing so is to
smoking, we should not expect a significant renovate the train station. The rest of the question
decline in number of deaths (because such provides premises in support of the claim that
accidents have always tended to have minor renovating the train station is the best strategy. The
consequences, decreasing the number of conclusion to the argument should be the claim
such accidents should not have a major effect about which the Mayor is trying to convince the
on number of deaths that result). Option (B), reader and which has been supported by the given
on the other hand, only provides that such premises. Option (A): While the Mayor makes this
deaths usually occur after everyone has gone statement, it is not the conclusion that his argument
to sleep. This does not help to explain why a leads to. This statement has already been agreed
decrease in smoking has failed to bring about upon and is therefore not the claim that the Mayor
a decrease in deaths caused by home fires. is trying to advance with the argument. Option (B):
Since this is the only option that does NOT This is the correct option. Not only is this the
help to resolve the paradox, it must be the
Mayor's motivation for making the argument, but it
right answer. Remaining options are irrelevant
is also the claim that the premises were designed to
as per the given situation.
support. Option (C): While the Mayor states that the
(Section-III)
number of jobs in Center City is increasing, there is
1. (C); the question explains that Braille is a system of
no claim made that Outerville residents should seek
raised dots that can be read by touch, and that a
work there. Option (D): While the Mayor suggests
new computer screen will replicate the Braille using
that driving is becoming more expensive than train
heated dots. The question concludes that anyone
travel, he does not claim that it is a better option
who is accustomed to reading Braille can easily
that should be used by all. This is not the Mayor's
adapt to use the computer screen. The question
motivation for making the argument. Option (E):
assumes without good reason that heated dots will
The Mayor is attempting to convince Outerville
work just as well as raised dots. Since you are
residents to renovate the train station. The
asked to identify a necessary assumption, you
argument is not meant to encourage people to
should focus on linking raised dots to heated dots.
move to Outerville, although that is the ultimate
Option (A): The question does not concern whether
goal of the plan.
systems other than Braille might work with the

Head Office: 127, Zone II, MP Nagar, Bhopal |+91-9111555433| www.legaledge.in Page 16 of 26
Marathon - I
3. (D); the method used by environmentalist doesn’t not the other way around. This is a mistaken
clarify, argue or attack the land developer’s point of reversal of that premise.
view. Hence options (A), (B) and (E) can be
eliminated. Our answer cannot be option (C) as it is 6. (E); based upon our knowledge of the real world — as
very extreme resolution which is not required. opposed to the crazy, mixed up LSAT world — we
Hence, correct answer is option (D). know that politicians often base their actions in
4. (D); option (A) this gives us no reason not to like the office on the results of opinion polls. If Astorga's
special treatment. In fact, I might like it even more campaign promises are crafted primarily from
now because maybe having all that extra support is opinion polls, who is to say that her actual actions
good. Option (B) This is comparing flat feet to "feet in office will not follow the same trend. In this
with unusually high arches." We know nothing assumption question, we must find the option that
about those kinds of feet. Eliminate. Option (C) Are would eliminate that possibility, so as to allow us to
these the children who wore the "special shoes?" If get to the question conclusion. Option (A): Just
the children who did NOT have corrected feet did because she will not be capable of carrying out her
wear the "special shoes" then this would be a good campaign promises, does not mean that she does
answer. However, it gives us nothing about the not intend to make the attempt to carry them out.
special shoes and that is what we want to talk The question is not addressing what she will
about! Option (D) this certainly gives us one reason actually accomplish, but what she intends to try to
to not like the treatment: the treatment is apparently accomplish. Option (B): The problem in the
pointless. There is absolutely no difference question argument has nothing to do with the
between wearing and not wearing the shoes so why accuracy or inaccuracy of opinion polls. The
wear the shoes? This definitely seems correct. question is merely assuming that she will not
Option (E) we only care about children who are flat- continue to rely on those polls — regardless of their
footed so this is incorrect option. accuracy — once she is in office. Option (C): The
5. (A); this question presents a traditional use of question conclusion is not concerned with the
conditional reasoning in a must be true question. It chances of Astorga being elected, but with what will
is stated that an increase in carbon dioxide leads to result if she is elected. Therefore, our assumption
an increase in the number of large bodies of water answer needs to deal with what will occur after the
with temperatures over 26 degrees Celsius to a election, not whether her campaign promises will be
depth of 60 meters. If the temperature of a large successful. Option (D): Whether or not Astorga has
body reaches this level, it is more likely that tropical strong opinions of her own does not affect the issue
storms will develop in that area. Therefore, an of whether she will simply follow opinion polls if
increase in carbon dioxide will make it more likely elected. Actually, if she has no strong opinions of
that there will be tropical storms in the areas of her own, that would aim more towards a conclusion
temperature increase. If this is true, it can be that she will simply follow the opinion polls if
inferred that there will likely be a larger number of elected. Therefore she would be telling the voters
storms overall: ..... Increase in CO2 → more Warm what she actually intends to do. Option (E): This is
Water → more storms likely. Option (A): This is the the correct option. Within the question, we have no
correct answer. This option combines the given link between what Astorga is promising and what
premises to reach a logical conclusion. If there is an she actually intends to do if elected. This option
increase in the number of areas with conditions that provides that link. Now that we know that she does
support tropical storms, there will likely be an not intend to do what she said she would, we can
increase in the total number of storms. Option (B): get to the conclusion that the voters are being
This is similar to a Mistaken Reversal of the first deceived.
premise. These temperatures simply make it more 7. (E); here the question is presenting a correlation
likely that there will be storms, they are not between retired persons who volunteer and better
necessary for storms to occur. Also, the premise resources, health, outlook and functioning. From
deals with water temperature, not air temperature. this correlation, the question concludes that there is
Option (C): The information in the question only a cause and effect relationship, in that the
allows us to compare the number of bodies of water volunteering is causing the better circumstances for
today with the number that there will be in the future these retired individuals. So, we have a cause and
if carbon dioxide levels continue to increase. There effect relationship in the question combined with a
is not enough information about past levels to flaw in the reasoning question. We are therefore
support this claim. Option (D): No information is going to try to find one of four things: 1) an alternate
given in the question about the ferocity of tropical cause; 2) that when the cause occurs, the effect
storms. All of the information presented deals with does not follow; 3) that when the cause does not
the number of storms, not their strength. Option (E): occur, the effect does; or 4) that it is actually a
In the question, it is an increase in carbon dioxide reverse cause and effect relationship. Option (A): It
that affects the temperature of the Earth's oceans, is likely true that the center has a selfish motive in
Head Office: 127, Zone II, MP Nagar, Bhopal |+91-9111555433| www.legaledge.in Page 17 of 26
Marathon - I
reporting this correlation. However, that motive, in the newly introduced one was actually the one was
and of itself, does not make the inference responsible for the great effects.
unwarranted. Just because the group reporting 9. (C); to support the claim that the new pesticide was
information has some type of bias, does not more effective than the old one, one must be able
automatically make the information untrue. Option to directly compare the results of the two on the
(B): There is no problem with interpreting "well- same type of trees. If trees that were treated with
being" to include these various factors. As opposed the old pesticide did not show a reduction in losses
to other undue assumptions made within LSAT of fruit to insects, then the conclusion that the new
questions, this correlation between the concept of pesticide was more effective than the more one is
"well-being" and the factors named is not strengthened. Option (A): If trees treated with the
unfounded. Further, it does not address the cause new pesticide did not show a reduction in losses of
and effect issue. Option (C): The fact that some fruit to insects, this would directly weaken the
retired persons who do not volunteer may be older official’s conclusion. Option (B): This would not help
than some who do volunteer does nothing to the official’s conclusion since by treating the trees
diminish the information that those who regularly with both pesticides, it is impossible to credit the
volunteer generally display fewer and milder effects reduction in losses of fruit to insects to the new
of agency. Option (D): Mental outlook is just one of pesticide and not the old. Option (C): This is the
the factors addressed by the question. This option correct option. If trees treated with the old pesticide
does nothing to address the volunteers' relative did not show a reduction in losses of fruit to insects
strength in the other areas. Also, once again, the but trees treated with the new pesticide did, and it
fact that growing older does not necessarily result supports the conclusion that the new pesticide was
in a change in mental outlook does not change the more effective. Option (D): This does not support
fact that the volunteers had a better mental outlook the official’s conclusion since it does not allow us to
than the non-volunteers. Option (E): This is the compare the effects of the old and new pesticide.
correct option. This demonstrates that the cause in Option (E): This does not support the official’s
effect relationship is in fact reversed. conclusion because it does not allow us to directly
8. (A); this is a classic weaken except question with a compare the effects of the old and new pesticide.
correlation-causation issue. In the question we are This is because the lack of reduction in losses of
told that a new pesticide (X) was used in a pear fruit to insects could be due to the properties of the
orchard. During the three years that this pesticide neighboring district, and not the pesticide.
was used, a lower proportion of fruit was lost to Moreover, by using the new pesticide, it does not
insects than during the three year period in which allow us to compare the effects of the old and new
another pesticide (Y) was used previously. The pesticide.
conclusion of this argument is that X is more 10. (B); Argument says - because of self interest there are
effective in the realm of losing fruit to insects. This quite a few motives evolved. Sometimes these
is coinciding of events, a correlation, could be motives affect some human actions and sometimes
explained in a multitude of ways based on the they affect all human actions. But those motives
evidence given. To weaken the causal argument, that affect all human actions are necessarily
we want to think what in fact weakens causation. evolved only from self interest, whereas those
However, we have to find that suitable option which affect only some human actions can be evolved
is strengthening the argument as the question is from either self interest or something else.
asking “except”. Option (A) is the correct choice as Therefore its concluded that self interest is the
it strengthens the argument by focusing on amount prime cause of human actions. Problem with the
of matured trees. Option (B) presents an alternative argument is just because some motives affect
cause, what if it was the case that during the period some or all human actions cannot make them the
of pesticide X being used, there was an insect primary cause and this is something we need to
assault campaign. That could have been what look for in the answer. Option (A) denies that an
caused the fruit to have not been harmed by the observation that a trait is common to all the events
insects, maybe the insects were lessened due to in a pattern can contribute to a causal explanation
this campaign that is unrelated to the pesticide of the pattern - trait is not common. Self interest
used. Option (C) is also an alternative cause. What doesn’t cause every human action. So not correct.
if the predatory birds that go after these insects are Option (B) takes the occurrence of one particular
becoming more common in the area? That could influence on a patterns or classes of events as
explain why insects are becoming less of a problem showing that its influence outweighs any other
to the fruit. Option (D) Alternative cause. What if the influence on those events. This seems to be
insects are losing their habitat and can no longer be correct. Self interest causes human action does not
a nuisance in the area. Option (E) Alternative mean its influence outweighs any other influence on
cause. Maybe the pesticide that was used prior to human actions. Option (C) concludes that a
characteristic of pattern or class of events at one
Head Office: 127, Zone II, MP Nagar, Bhopal |+91-9111555433| www.legaledge.in Page 18 of 26
Marathon - I
time is characteristic of similar patterns or classes inspiring, but rather that Vierne’s music is religious
of events at all times. This would be true if we because it seems to be divinely inspired (whereas
assume that self interest affects all human actions previously “religious” meant taken from religious
since its affects just one human action. Since we texts). Option (B): This is the correct option. As
are not discussing that therefore it is incorrect. discussed above, the author uses the notion of
Option (D) concludes that because an influence is “religious” in two different ways in the question.
the paramount influence on a particular pattern or Option (C): Whether or not other organ music is
class of events, that influence is the only influence divinely inspired is not a part of the author’s
on the pattern or class of events. We are not argument. Option (D): This tricky option also
assuming that self interest only affects all actions. represents an uncertain use of a term, but the term
We wont to disapprove that self influence is not the “symphonies” does not change meaning in the
only cause. So not correct. Option (E) undermines argument (although it is used twice, it means
its own premise that a particular attribute is present “musical compositions” both times). Option (E): We
in all instances of a certain pattern or class of cannot know that the author believes all organ
events. It doesn’t help to prove what is expected. music to be religious. All we know is that the author
11. (E); this is a method of reasoning based question. The believes Vierne’s works are religious.
author starts by explaining a general argument 13. (C); In this question, Charles points out that during
made by "many people." We know that when "many recessions unemployment generally rises, which
people" or "some people" argue something, the means that fewer people are commuting to jobs. He
question author will almost always make a argues that, because fewer people are traveling to
counterargument against those people. That is the jobs, cars emitting pollutants are used less, and
case here, as the author provides a specific concludes that air pollution due to automobile
example to dispute the general argument exhaust decreases. Darla wonders why we should
presented. Option (A): At no point does the author think that air pollution (from cars) would decrease,
appeal to any type of scientific authority. Just as in because during a recession fewer people can afford
Must Be True questions, you must be able to new cars, but as cars age they pollute more. Darla
support your answers in Method of Reasoning with makes a legitimate point, because even with fewer
something that specifically occurs in the question. cars on the road, pollution could increase if the cars
Option (B): No evidence is provided to support the are "dirtier." Charles' argument is based on
original argument. There is no way for the accuracy incomplete evidence, and many issues could
of non-existent evidence to be questioned. Option damage his conclusion. Darla brings up an
(C): This question is not dealing with a natural additional consideration, and you should realize
phenomenon. It is addressing the issue of not being that Darla has provided the critique you will likely
able to observe stars because of the man-made need, and proceeds towards the options. This
lights in cities. Further, the author provides a question asks you to describe the relationship of
solution to a problem, not a scientific explanation to Darla's response to Charles' argument, so you
a phenomenon. Option (D): There is no distinction should focus on the fact that Darla introduces a
of terms made, or necessary in this question. new interpretation of the evidence Charles has
Option (E): This is the correct option. The general presented. Option (A): Darla does not attack
claim is that interference from lights is inevitable. Charles' premises; she merely adds the entirely
The counterexample is the town of Sandsville, new consideration of whether the cars on the road
which has restricted unnecessary lighting and might be "dirtier" during a recession. That shows
installed special street lamps that direct all their that Charles' premises may not be sufficient to
light downward. prove his conclusion, but it is an attack on the
12. (B); Although not explicitly written out, the music critic’s argument, not the premise. Option (B): This choice
conclusion is that Vierne’s works should be states that if Darla's claim is true, then Charles'
considered religious music. This is based on the conclusion is false. However, Darla's claim does not
fact that, even though Vierne’s works are prove Charles wrong. Introducing additional
apparently not based on familiar religious texts (like considerations generally reduces a "strong"
Handel’s works), hearing Vierne’s music in a conclusion to an "uncertain" conclusion; it does not
particular setting invokes emotional feelings of prove the conclusion false, so this choice is
divine inspiration. The flaw here is in the shifting incorrect. Option (C): This is the correct option.
definition of “religious.” The first usage of the word Darla's additional consideration of whether the cars
is in reference to source materials (“religious on the road would be "dirtier" challenges the notion
texts”), while the second, which the author uses to that simply having fewer cars on the road would be
make his/her conclusion is about religious/“divine” sufficient to prove that pollution would decrease, so
inspirations (listener’s emotional response). This is she weakens the support that Charles' conclusion
known as an uncertain use of a term. Option (A): receives from his evidence. Option (D): Darla
The author never claims that all religious music is questions whether Charles' conclusion is true, so
Head Office: 127, Zone II, MP Nagar, Bhopal |+91-9111555433| www.legaledge.in Page 19 of 26
Marathon - I
this response that claims that she supports his environmental factors is to breed the birds in
conclusion is contrary to the question, and captivity and return them to the wild later.
incorrect. Option (E): Darla does not commandeer One way to approach the Question is to diagram:
Charles' reasoning to demonstrate an absurdity; ..... Condors survive → breeding population must
instead, Darla adds a consideration that shows that be increased; and
Charles' argument may not be sufficiently complete. ..... Breeding population increased → most eggs
14. (D); this question asks you to identify a necessary must hatch.
assumption in Charles' argument. Since you know Putting the two statements together:
that Charles ignores additional factors, you should ..... Condors survive → breeding population must
focus on that when you review the choices. Option increase → most eggs must hatch
(A): If people who have never been employed keep ..... Condors survive → most eggs must hatch
up their driving levels during a recession, that does In plain English, if the condor is going to survive in
not help prove that fewer cars would be on the the wild, most of its eggs must hatch. The Question
road, or that pollution would decrease, so this Stem asks which one of the following Options is
unhelpful response can be quickly eliminated. most supported by the information above. This is a
Option (B): Since Charles does not argue that Must Be True type of question because it asks what
overall air pollution would decrease, and restricts inferences can be made from the information that is
him to pollution caused by automobiles; a response contained in the Question. Whenever you have a
about the relative amount of pollution that SufNec Question with a Must Be True question
automobiles contribute is irrelevant to his argument, type, look for an Option that contains the contra-
so this choice is wrong. Option (C): The effect of positive of the reasoning contained in the Question.
this choice upon the argument is unclear. If most The pre-formed answer is the contra-positive of the
people employed do not use public transportation, reasoning in the Question. The contra-positive is:
which makes it more likely that, recessions would ..... Most eggs hatch → Condors survive
affect drivers. However, if most of the people In plain English the contra-positive is if most eggs
employed did use public transportation, it could still do not hatch, then the condors will not survive.
be true that a large number of those employed Option (A) says that the condor will eventually
drive, so recessions could still affect drivers. This become extinct in the wild. If most of the eggs do
choice fails the negation test, and is incorrect. As a not hatch, this is true, according to the reasoning in
point of clarification, Charles does need to assume the Question, but it assumes that most of the eggs
that some of those employed sometimes do not use do not hatch. The Question gives no indication as
public transportation, but that assumption does not to the likelihood of the eggs hatching or not. This
need to be any more extreme. Option (D): This is Option is not the contra-positive of the reasoning in
the correct option. Charles needs to assume that the Question, and therefore, it is not the correct
additional factors do not counter the possibility that Option. Option (B) states that the best way to save
fewer people need to drive to work. If the reduction the condor in the wild is to breed it in captivity. This
in driving to work were offset by an increase in is a very attractive answer, but the Question
driving for other reasons, Charles' conclusion would discusses that breeding in captivity is "one possible
not follow, so it is critical that Charles assume there way" to eliminate effects of environmental factors
is no countering increase of automobile use for that impact the breeding of the condor. It may very
other reasons. Option (E): Charles does not need to well be true that the best way to save the condor is
assume that more people who drive than people to breed them in captivity, but that information is not
who do not drive lose their jobs during a recession. contained in the Question and requires additional
As long as some people who drive lose their jobs, information. Additionally, it is not the contra-positive
the recession could have the effect he proposes on of the reasoning contained in the Question, and
drivers, even if most people were not driving, so therefore, it is not likely to be the correct Option.
this choice is wrong. Option (C) states that it is almost impossible to
15. (D); this Question contains some conditional reasoning eliminate all of the environmental threats to the
statements (Sufficient and Necessary). The first eggs of the condors. The Question does not
such statement is the initial sentence: if the condor discuss eliminating all of the threats – just reducing
is going to survive in the wild, then the breeding them as much as possible. This inference goes
population must be increased. A second conditional further than what is stated in the Question and it is
relationship presented in the Question is that if not the contra-positive of the reasoning contained in
there is going to be any increase in the breeding the Question. Therefore, it is not likely to be the
population, then most of the eggs must hatch. The correct Option. Option (D): This is the correct
likelihood of the eggs hatching is impacted by option, as it states that if the condor eggs do not
environmental factors. The author suggests that hatch, then the condor as a species will not survive
one possible way to eliminate the effects of the in the wild. This is an exact reproduction of the
contra-positive as stated above. Option (E) states
Head Office: 127, Zone II, MP Nagar, Bhopal |+91-9111555433| www.legaledge.in Page 20 of 26
Marathon - I
that the most feasible way to protect the condor larger than the current population. --> Necessary
from extinction is to increase egg production. The condition is not met.
Question does not discuss feasibility at all, only Analysis:
possibility. Additionally, the Question does not Fact 2 means previous cheetah population needed
discuss egg production; it does discuss getting to survive a natural disaster was too small.
more eggs to hatch, which should not be confused Fact 3 means the current cheetah population is
with egg production. Option (E) introduces two new even smaller that in the past. It means the current
concepts to the discussion and therefore it should cheetah population is not enough to survive a
be eliminated. natural disaster.
16. (E); this is a parallel reasoning question. This question Fact 4 means the current cheetah population can’t
provides a fairly simple argument: become larger because there is not enough African
Premise: ..... Increasing steel imports would hurt grassland. Fact 4 clearly states that the cheetah
the steel industry. population is not increasing, at least in the short
Premise: ..... The present government would not do term. Combine fact 2, 3, 4 we can say that in the
anything to hurt the steel industry. short term, the wild cheetah population will be
Conclusion: ..... Therefore the government will incapable of surviving a natural disaster in the
continue to restrict those imports. Because this African grasslands. Thus, only option (D) is correct.
argument is on the simpler side, we will more likely 18. (B); the give principle is – work of art must satisfy 2
be able to double the conclusion and quickly necessary conditions: (a) originality and (b) far
eliminate at least some of the wrong options reaching influence upon artistic community. Option
presented. Again, the author concludes that the (A) lacks anything about far reaching influence.
government will continue to restrict steel imports. Option (B) is the correct answer satisfying both the
This is an absolute conclusion, so the option which conditions. Option (C) presents originality and
parallels the author’s argument should provide a profound impact. That is different from far reaching
similarly absolute conclusion. Option (E): This is the influence. Option (D) is a cause and effect
correct option. Option (E) is the only choice which argument. The art needs originality and far reaching
displays valid logic and an absolute conclusion. The influence to be truly great. Not the other way
argumentation, as shown below, provides a solid around and also this does not talk about far
parallel: reaching influence. Option (E) is a cause and effect
Premise: ..... Bright floodlights would make the argument. The art needs originality and far reaching
telescope useless. influence to be truly great. This can be eliminated
Premise: ..... The astronomy department wouldn’t as extreme condition.
do anything to make the telescope useless. 19. (A); the given condition is – a particular crater was thought
Conclusion: ..... The astronomy department will not to have been made by a meteorite that caused
support the floodlight idea. Incorrect option (A) ends mass extinctions in the late Mesozoic era.
with a conclusion that is slightly less absolute: “we Professor Robinson does not believe the impact
can expect…” Additionally, this choice reflects a that caused this crater was to blame for the mass
mistaken negation, flawed reasoning which cannot extinction. This is because molten rocks reflect the
parallel the valid reasoning found in the question. Earth’s polarity at the time they crystallize, and the
None of the other choices’ conclusions parallel the rocks around the crater are the reverse of the
absolute conclusion found in the question. Option polarity of the Earth at the time of the extinction.
(B) provides, “there is no need…,” which doesn’t tell The question is followed by an assumption X
us much about whether such regulations will be question, which means that the all of the answers
established. Incorrect options (C) and (D) both end except for one will provide an assumption on which
in limited, rather than absolute, conclusions: “they the author’s conclusion is based—that the meteor
will probably result…,” and “will probably not accept that caused the crater was also responsible for the
stock…”. mass extinction. The correct option will provide an
17. (D); the question can be summarized as: assumption on which the author’s conclusion does
Fact 1: Wild cheetahs live in the African grasslands. not rely. Option (A): This is the correct option,
It is an necessary condition. because the author did not need to assume that the
Fact 2: Previous estimates of the size that the wild crater was more than sufficient size to cause the
cheetah population must be in order for these mass extinction—it could have been just
animals to survive a natural disaster in the African sufficient—that, of course, would have been
grasslands region were too small. enough. To confirm this as the right option, we can
Fact 3: The current population barely meets the take it away (by logically negating it) and note
previous estimates. whether or not the argument is weakened: “The
Fact 4: At present there is not enough African crater was not more than sufficient.” Again, since it
grassland to support a wild cheetah population could have been just enough, this negated version
would not hurt the argument. Option (B): This is an
Head Office: 127, Zone II, MP Nagar, Bhopal |+91-9111555433| www.legaledge.in Page 21 of 26
Marathon - I
assumption that the author needs, because the argument has nothing to do with the example of
conclusion is based on the inconsistent polarity music, but rather its discussion of paintings. Option
between the crater site and the earth at the time of (C) This is an example of an option that at first
the extinction; the argument assumes that the area glance seems profound, but upon reflection, really
around the crater reflects the polarity at the time does not say anything useful at all. What are these
that the crater was formed. A significant delay in aspects common to both music and painting that
crystallization would thus weaken the argument. the argument fails to bring out? There may be
Option (C): Since the author’s conclusion is based many, and indeed, can be anything that music and
in part on the assumption that the rocks around the paintings have in common, but even if brought out,
crater reflect the polarity at the time of the impact, if such would not remedy the flaw in the argument.
the rocks at the location were melted by some other Option (D) it essentially criticizes the argument for
event, that would weaken the argument. Thus the relying too much on the metaphor of "reading," so
author’s argument does assume that no other event much so that the metaphor itself assumes the
melted the rocks, so this choice cannot be the conclusion to be established. However using the
answer to this Assumption except question. Option metaphor of reading in no way causes the
(D): The author assumes that the surrounding rocks argument to assume the truth of the conclusion.
were melted by the impact—if something else Rather, the argument still focuses on the lack of a
caused the rocks to melt, that would weaken the set sequence in which things are presented in a
author’s argument, which depends on linking the painting to conclude that paintings lack a temporal
melted rocks to the timing of the impact (and the dimension. Option (E) is the correct option. It
mass extinction). Option (E): Again, the author’s rephrases our objection in more eloquent terms.
argument is that the impact happened at some time Hence, when viewing a painting, there exists some
other than the extinction—this is based on the path that the eye takes, and therefore, there exists
inconsistent polarities. If we negate, or take away, a temporal dimension in viewing a painting as the
this assumption, we get: The extinction would not eye moves along this path.
have occurred soon after the impact”. If that is the 21. (D); there are two studies to deal with in this question.
case then that would weaken the author’s The first involves the relative earnings of men and
argument, which is based on the assumption that if women who are employed full time. The second
the meteor had indeed caused the mass extinction, involves the relative earnings of all employed men
the two events would have happened around the and women. The relative earnings for all employed
same time. women are much lower than the relative earnings
20. (E); the argument in this question concludes that music for full time employed women. In such a situation,
has a temporal dimension, while paintings do not. you should look to see what would likely be the
This is supported by the fact that music is factor in one survey that would cause such a
presented as a set sequence of sounds, whereas deviation. Here, the one thing that does not seem to
paintings are presented as a whole to the viewer, be addressed is the fact that part-time workers
and the viewer has no set path to follow with his were included in the second study. Therefore,
eyes. The obvious retort to this argument is "What chances are the correct answer will involve part-
about the path that the viewer creates for himself time workers. Option (A): The discrepancy we are
with his eyes when he views the painting? How is trying to resolve is based upon current salaries, not
viewing the painting first from the corners, then on the changes in salaries through the years.
inwards not a temporal sequence?" As such, even Therefore, any answer focused upon those
though the viewer creates his own path while changes will do nothing to resolve the current
viewing the painting, the path that he creates discrepancy. Option (B): This similarity between
supplies the temporal dimension. The only men’s and women’s salaries in similar full time
difference, therefore, is who creates the path—with positions would be factored into both studies, so
music, it is the composer; with paintings, it is the therefore would not have any effect on the
viewer. Nevertheless, in both cases, there remains differences between the two studies. Option (C):
a temporal dimension. Option (A) It mentions the This would be a good option to explain changes in
passage of time as a red herring. However, women’s relative earnings from year to year, but it
remember what the argument is about—the does nothing to explain the differences in the two
absence of a set sequence or path in which the art studies regarding current earnings. Option (D): This
form is presented. Immersion in the experience of a is the correct option. Here we finally find our option
painting so that one loses track of the passage of that takes the part time workers into consideration.
time has nothing to do with the rest of the question. Knowing that more women than men are part-time
Option (B) The question is mainly dealing with workers and part-time workers earn less than full
paintings, and only uses music as an example of an time workers, explains the discrepancy between the
art form that is allegedly distinct from paintings in first study and the second study. Option (E): We are
that it has a temporal dimension. The flaw in the concerned only with the differences between the
Head Office: 127, Zone II, MP Nagar, Bhopal |+91-9111555433| www.legaledge.in Page 22 of 26
Marathon - I
two studies in Naota. Any information regarding cigarette smokes have a greater chance of heart
differences in other countries is going to do nothing disease than non-smokers. Also, this option is
to resolve the discrepancy between the two Naota missing a sub-conclusion, which the question has.
studies. Option (C): This answer is missing a subsidiary
22. (D); given condition can be framed as : Some claim conclusion, which the question has. Moreover, the
island goats have evolved to be smaller, in order to conclusion of this answer also does not match that
increase their number and ensure a diverse gene of the question: it should conclude, instead, that
pool. But there is only one way goats would evolve "people who want to avoid dental problems should
(and make the trait of being small more common), not drink coffee”. Option (D): This option is not
and that would be for smaller goats to have greater missing the basis for comparison because it states
success reproducing. So, the biologist disagrees that "people who do not exercise regularly have a
with the claim that the goats have evolved to shorter life expectancy than those who exercise
increase their numbers by becoming smaller-- regularly". Moreover, this choice lacks a sub-
evolution doesn't work that way--the individual conclusion. Finally, the conclusion does not match
goats don't sense the need for greater numbers and that of the question: it should instead state that
respond with smaller offspring. Instead, what has "people who wish to live a longer life should
lead to smaller goats on average must be some exercise regularly”. Option (E): This is the correct
factor other than evolutionary pressure. This is option. Most people who exercise regularly can
provided by correct option (D), which states that handle stress.
evolutionary pressure would not cause smaller Sub-conclusion: Cause ..... ..... ..... ..... ..... Effect
goats to become more common. Option (A) is an Exercising Regularly ..... = ..... decrease chance of
Opposite Answer: take a look at the wording--it is a being overwhelmed by Stress. Thus, to handle
perfect logical opposite of correct answer (D). stress better, one should exercise regularly. Again,
Option (B) is incorrect because greater reproductive this is an incorrect reasoning since it is missing the
success could lead to a greater frequency of small basis for comparison: if everyone who does not
goats in island populations. Option (C) if true would exercise regularly can handle stress, for example,
not support the biologist's view and would appear to then it seems that exercising regularly is actually
present a paradox, because smaller goats are more bad for handling stress, and the sub-conclusion and
frequent in island populations. Option (E) provides conclusion of this argument will no longer stand. All
that reduction in the average size of the population the parts of this argument match those of the
is necessary. As you point out though, this is question, and it commits the same flaw, thus it is
irrelevant, because the question is whether the correct answer.
evolutionary pressure could have led to that size 24. (C); Mr. Nance states that Ms. Chan said that she has
reduction. retired from Quad Cities Inc., but that he has
23. (E); The question of this problem contains a flawed overheard one of her colleagues say that Ms. Chan
reasoning because it is missing the basis for will be on business trips for much of the next year,
comparison: so he concludes that Ms. Chan or her colleague is
75% of strict vegetarians reach 50 without lying. Mr. Nance is ignoring the incredibly obvious
developing heart disease. fact that Ms. Chan could simply work for another
Sub-conclusion: Cause ..... ..... ..... ..... Effect company next year, or attend to her own business
Vegetarianism ..... = ..... avoid heart disease. affairs. Since you are asked to identify the flaw in
Therefore, to reduce risk of heart disease, one Mr. Nance's reasoning, you should focus on the fact
should not eat meat. This is, however, incorrect that he has ignored the many interpretations of the
reasoning since we do not know what percentage situation in which no one is lying. Option (A): Mr.
of non-vegetarians reach 50 without developing Nance's argument is not based on hearsay; rather,
heart disease. If 75% of non-vegetarians reach 50 his argument is based on an overly extreme
without developing heart disease, for example, then acceptance or rejection of hearsay. If Mr. Nance
the sub-conclusion of this argument is no longer had concluded that Ms. Chan is definitely lying, he
valid, neither is the final conclusion / would be basing his argument on hearsay;
recommendation. Thus it is an incorrect reasoning. however, Mr. Nance is just as open to rejecting the
Option (A): The reasoning in this option is sound, hearsay as to accepting it. Option (B): Mr. Nance
so this cannot be the right option. The conclusion of does not criticize Ms. Chan, as he does not identify
this argument also does not match that of the the exact source of the "lying." Furthermore,
question: it should say something to the effect of asserting that a claim is not true, or that someone
"To avoid traffic accidents, one should drive below has lied, is in fact a criticism of the "claim." A
the speed limit". Finally, this argument is missing a criticism of the person would involve designating
sub-conclusion, which the question has. Option (B): someone as a "liar." Option (C): This is the correct
The reasoning in this option is sound as well. It is option. The equivocal language referenced in this
not missing the basis for comparison: it states that option concerns use of the term "retired." Mr. Nance
Head Office: 127, Zone II, MP Nagar, Bhopal |+91-9111555433| www.legaledge.in Page 23 of 26
Marathon - I
apparently believes that his definition of retirement for you to notice that this choice is very probably
should dictate the retirement activities of others. the correct response, even if your analysis is no
While Nance may believe that retirement means deeper. Option (C): The argument is not based on
the ambiguity of any of the terms. You should avoid
living on a golf course, it may be that Mr. Chan is
choosing such a response when you simply find the
simply retiring from his current position while question confusing. Frequently, confusing stimuli
continuing to work. Option (D): Since this response are accompanied by choices about circular
would actually mean that Ms. Chan has lied to her reasoning, ambiguous terms, and impossible
colleague, this response could not describe a flaw. reasoning in the hopes that confused test-takers
Even if Mr. Nance did not consider this possibility, will basically choose the suicide option for such
the possibility that Ms. Chan lied only supports the questions. If you are confused, eliminate these
types of choices. This choice is incorrect. Option
idea that someone is not telling the truth, so this
(D): It is unclear what the third specific belief is,
choice is wrong. Option (E): Actually, assuming that unless it concerns comparing the numbers of
someone has superior character on the basis of a people who hold certain beliefs. It is actually not a
long and loyal service is itself a logical flaw, and it is flaw to use information about two different beliefs to
not a flaw to refuse to infer things that do not make conclusions about certain third beliefs, as
logically follow. This choice is wrong. Furthermore, long as the information is used in a consistent,
Mr. Nance never claims that Ms. Chan is the one logical manner. Since it is not the goal, but rather
the method that is flawed, this choice is wrong.
who is lying.
Option (E): The argument contains no contradiction,
25. (B); The question concludes that more people believe and this choice is for the testers who could not
elected officials should resign if indicted that believe understand the 50% and 35%. If you chose this
they should resign if convicted, and argues that on answer, you probably realized that the conclusion
the basis that 50% of people believe an official had to be false, because if 50% of people believe
should resign if indicted, whereas 35% believe one politicians should resign if indicted, then at least
50% of people have that same belief about
should resign only if convicted. If you keep your
conviction. However, you probably did not realize
Conditional wits about you, you can escape the that the 35% is described by "only if," so is not
somewhat confusing nature of the question by contradictory.
simply observing that since the 50% is (Section-IV)
accompanied by an "if," while the 35% is 1. (A); Option (A) is correct answer. Siuzdak counters
accompanied by an "only if," the percentages Janeck's statement by saying that the experience of
cannot be compared because they are about running a major corporation is a valuable
logically different types of conditions. experience to run for office and that he is qualified.
This means that Siuzdak interpreted Janeck's
If not, you will need to diagram the question so that
remark to be suggesting that Stevenson was
you can clarify the issue: unqualified for the office of governor. Option (B)
Premises: ..... 50% believe "Indicted ..... = ..... Mr. Janeck does not say this. He does imply that
Should Resign" many voters would not vote for a businessman with
..... ..... 35% believe "Resign ..... =..... Convicted." no political experience. What about a Lawyer with
Conclusion: More believe that a politician should no political experience? This statement says No
resign if indicted than believe a politician should Candidate, regardless of previous work/career
resign if convicted. First of all, you should recognize experience, has been elected without political
once again that you cannot make the comparison in experience. Option (C) Mr. Janeck says the exact
the conclusion, because you do not have opposite. Option (D) the issue is experience here.
information about what people believe should occur Not how profit chasing affects the character of a
if someone is convicted, because conviction was person and makes him unsuitable for government
never presented as a sufficient condition. Secondly, office. Option (E) is incorrect because she
that conclusion is absurd, since everyone who has overlooked voters completely and responded to Mr
been convicted has by definition been indicted (that Janeck as if he had said the following: "A
is common knowledge and you are supposed to businessman with no political experience should
realize it). Everyone who believes a politician not be elected to such a responsible public office."
should resign if indicted by definition believes that 2. (D); option (A) higher income group will not lend support
the politician should resign if convicted. The LSAT as they will end up paying more in a flat tax regime
test-writers are simply using numbers that seem to than under progressive tax rate. Option (B) the
accompany their reasoning quite well in the hopes increase or decrease in tax revenue to the
that you will have a hard time disbelieving their government has not been mentioned. No numbers
fundamentally absurd and flawed reasoning. to support that the revenue collected under flat tax
However, the easiest way to get this question system will be higher. The ratio of lower income
correct is simply to observe the illogical comparison and higher income group will also be a factor, which
of a sufficient condition to a necessary position, and has not been disclosed. Option (C) not necessarily,
proceed to identify that flaw. Option (A): This choice maybe the flat tax will be fixed at rates presently
references an overgeneralization, but the question prescribed for the lower income group. Their
involved a necessary-sufficient error. This choice is position will be unaltered. Option (D) The
wrong. Option (B): This is the correct option. Simply progressive tax system does not always tax
focusing on the "if" and "only if" should be sufficient thehigher income group more. The actual tax in

Head Office: 127, Zone II, MP Nagar, Bhopal |+91-9111555433| www.legaledge.in Page 24 of 26
Marathon - I
comparison to their income may be lower than the “truth”, this conclusion is shown analogous to
tax rate being levied on the lower income group. “water”. Hence only option (C) is the correct
Option(E) possess same logic as option (C). The answer.
lower income group may not enjoy a different 11. (E); the main point can be inferred directly from option
situation but the higher income group may end up (E). Remaining options are irrelevant hence
paying a higher tax rate than the effective tax rate eliminated.
under the progressive system. Hence option (D) is 12. (D); the given argument comprise of :
the appropriate answer. Conclusion: the Daily Bugle is owned by an out-of-
3. (D); the question is asking about "misuse of confidential town business syndicate that couldn't care less
information" and hence we can eliminate options about the people of Gothan City.
(A), (B), and (C). By clarifying option(A) "all Evidence: Daily Clarion is the only real voice of the
persons" there is nothing a person who is people Gothan City. The fundamental thing about
concerned about his/her misusing his/her question like this is if you inadvertently take
information. So is option (B) as it describes the conclusion for evident, you will miss the point.
depth of harm/abuse, which again is of little Therefore, for refuting means to prove to be false
importance/interest to the person who is concerned. the argument, we must select answer that denies
Eliminating option (C) is easy. We are left with the conclusion. Option (A) does not deny anything
options (D) and (E). What interests more for a about Daily Bulge. Option (B) does not deny Daily
person concerned about "misuse" of his confidential Bulge. Option (C) does not deny Daily Bulge.
information - US Laws - may be NO. Knowing all Option (D) refuses the fact that Daily Bugle is
the US LAWS would of no-help for someone who is owned by out-of-town business syndicate. If the
compliant with laws. Also, knowing "appropriate Editor-in-chief and all other editorial staff have lived
circumstances" when the information be released to and worked in Gotham City for ten years, then we
authorized personal is important. So option (D) is cannot really say they less about the people of
the correct answer. Gothan City. Option (E) it’s not about how many
4. (E); While cameras, stereos, and video games arguably years Daily Bulge has been publishing. Hence
rank as ’high technology’, frozen foods certainly do correct answer is option (D).
not. Thus, option (E) fails to strengthen, and is 13. (E); the main argument is depletion of earth’s resources
correct. Remaining options are strengthening hence at very fast rate. As mentioned in the second
eliminated. sentence of the passage only option (E) satisfies
5. (B); only option (B) best summarizes the passage. the mandatory assumption. Hence (E) is the
Option (A) passage only says Johnson's text is still answer.
guilty of its own forms of distortion hence discarded. 14. (B); option (A) if new resources are being constantly
Options (C), (E) and (D) are irrelevant and out of discovered, then there is no need for the US to
scope. keep a check on their resource consumption. This
6. (C); the main argument has 2 parts - Premise: Law option weakens the argument. Option (C) this
requiring companies to offer parental leave will option teaches an important lesson: More countries
harm the nation’s business competitiveness; need economic development- There is no reason to
Conclusion: Companies must set their own parental believe so. Do not use your prejudice in this option
policies. Options weakening the argument are: that US does not need more development as
option (A): irrelevant, this doesn't say that compared to other countries. Approach the
strengthening the family means more a competitive question as if United States is Country X which you
nation; option (B) if many offer some form of leave, have never heard of in that case you can tell that
then this could strengthen the fact that comps there is no reason to prove that other countries
should set their own policies; option (D) irrelevant; need more development. Option (D) other countries
option (E) is again irrelevant. Option (C) is the have put a check on the resource consumption but
correct answer because if some with the most does it imply the United States also needs to do so
competitive businesses have strong regulations, . It is possible that other countries 's present level of
then there must be something flawed in the consumption is 50 times that of US .In that case by
argument. US putting a check on its consumption does not
7. (C); we are given A -> B, B -> C, and C -> D, and told help World's cause too much. Option (E) since its
they're all true. Since they're all true, you can say A been conserving resources for several yrs it might
-> D. Then by the law of contra positive, ~D -> ~A. already be judicious in using its resources is what
The answer is (C). this option suggests which is not suggested
8. (A); only option (A) can be easily inferred while anywhere in the argument and can’t be verified, if it
remaining options are irrelevant as per situation. were the case the world resources would not have
9. (A); option (A) is the answer. "It is ironic that the been threatened. Option (B) since US takes 1/3rd
innocent passenger should suffer for the driver's of the share of consumption, this indicates that US
carelessness, while the driver often suffers only is one of the major shareholder in consumption and
minor injuries or none at all." What if the driver was maintaining current levels is the least it can do to
not at fault? i.e. another car caused the accident. prevent the resources from depleting at a faster
For the reasoning to be true, most automobile rate. This is the only option which strengthens the
accidents must be caused by the driver in which an argument to some extent.
occupant is killed. A states this. Remaining options 15. (C); in option (A) the facts she presents do not support
do not go with the reasoning of the passage. her conclusion that Senator Frank is soft on crime. -
10. (C); From the last line of the passage we can come to She does not present any such facts. In option (B)
main conclusion that despite of various factors she assumes without proof that crime is the most
author is coming back to point of following the important issue in the upcoming election. - It is the
Head Office: 127, Zone II, MP Nagar, Bhopal |+91-9111555433| www.legaledge.in Page 25 of 26
Marathon - I
issue which she is concerned with. In option (C) from driving up property values, Proposition 13 has
she argues in a circle, using an unsupported saved homeowners thousands of dollars in property
assertion to dismiss conflicting evidence. - I agree, taxes. - "preventing inflation" is additional
she states that the latest law must be soft because information. Option (D) If Proposition 13 is not
he is not a strong supporter of crime. She does not repealed; identical properties will continue to be
prove so. Hence it is the correct answer. Option (D) taxed at different rates. - Properties is
she attacks Senator Frank on personal grounds generalization of houses. We cannot infer it from
rather than on his merit as a political leader. - The the passage. Option (E) Proposition 13 has
issue is related only to war on crime. Option (E) in benefited some homeowners more than others. -
deciding not to vote for Senator Frank, she fails to I'm not sure but it seems to be incomplete and "has
consider issues other than crime. - Irrelevant. benefited" is not clear here for me. Hence option
16. (C); the author in the above passage is condemning the (B) is the correct answer.
television coverage on election campaigns and only 18. (D); option (D) can be deduced from sentence- "But
option (C) is providing appropriate solution to the what does the company think sailboat hulls used to
issue highlighted in the passage. Option (A) is be made of?”. Hence D is the correct answer.
talking about number of hours in terms of length of Option (A), (B) and (C) are irrelevant while option
election campaigns which is not our concern, the (E) is out of scope.
main topic is regarding its television coverage. 19. (A); the main conclusion of author is – highlighting the
Hence too vague and can be eliminated. Option mania for high technology by a leading chemical
(B), (D) and (E) are irrelevant options. company. Only option (A) weakens the conclusion
17. (B); option (A) proposition 13 is unconstitutional as it is talking about good quality and advantage of
because it imposes an unequal tax on properties of product produced by high technology. Remaining
equal value. - "unconstitutional" is an additional options are strengthening the conclusion hence
information. Option (B) if proposition 13 is repealed, eliminated.
every homeowner is likely to experience a 20. (D); the main quality the teacher wants to imbibe in
substantial increase in property taxes. - no visible student is patience i.e option (D).
contradictions. Option (C) by preventing inflation

Head Office: 127, Zone II, MP Nagar, Bhopal |+91-9111555433| www.legaledge.in Page 26 of 26

You might also like